SóProvas



Questões de Logaritmos


ID
93295
Banca
CONESUL
Órgão
CMR-RO
Ano
2008
Provas
Disciplina
Matemática
Assuntos

Utilizando os valores  log10  2  = 0,30e log10 3 = 0,47 assinale a alternativa que corresponde a log10 12

Alternativas
Comentários
  • log10 12=log 10 (3.2^2)= log10 3+ log 2^2log10 3+ 2.log10 2=0,47+2.0,30=1,47o expoente pode vir multiplicando porque estamos trabalhando com logartimo,e os outros dados forma dado no problema, espero que ajude.
  • No enunciado da questão quando diz 2=0,3 está errado. Na verdade é log 2 = 0,3

    O colega acima fez certinho, mas na hora da última soma se enganou, pois a resposta é letra C.

  • O enunciado não está muito claro, mas fica subentedido que log102 = 0,3

    Se log102 = 0,3 e log103 = 0,47

    então log1012 será:

    log1012
    log10 4.3
    log10(2)2.3
    2log102 + log103
    2(0,3) + 0,47
    0,6 + 0,47
    = 1,07
  • 2*2*3 = 12

    log 2 + log 2+ log 3

    0,3+0,3+0,47 = 1,07

  • log12

    log(3.4)

    log3+log4

    log3+log2^2

    log3+2.log2

    0,47+2.0,30

    o,47+0,60=1,07

  • Depois que aprende logaritmos, já era:

    log de 2 = 0,3

    log de 3 = 0,47

    log de 12 = ???

    _______________________

    log12 = log 2*(2) x 3 = log 2*(2) + log 3

    = 2 log2 + log3

    = 2 * 0,3 + 0,47

    = 0,6 + 0,47

    = 1,07


ID
184873
Banca
CESGRANRIO
Órgão
Petrobras
Ano
2010
Provas
Disciplina
Matemática
Assuntos

Quando os alunos perguntaram ao professor qual era a sua idade, ele respondeu: "Se considerarmos as funções f(x) = 1 + log3x e g(x) = log2x, e a igualdade g(i) = f(243), i corresponderá à minha idade, em anos." Quantos anos tem o professor?

Alternativas
Comentários
  • logb a = X é o mesmo que bx = a

    f(243) = 1+ log3 243 => f(243) =1+ (3X =243) --> x=5 => f(243) = 6

    g(i) = f(243) então

    g(i) = log2i=6

    i=64

  • Gabarito= 64

    g(i)=log2i
    f(243)=1+log3243

    Obs= 243=35    

    log2i=1+log3243                            
    log2i=1+log3(3)5
    log2i=1+5log33
    log2i=1+5.1
    log2i=6
    i=2
    i=64

  • f(x) = 1 + log[3] x e g(x) = log[2] x    Se g(i) = f(243) é só trocar os valores.
    log[2] i = 1 + log[3] 243  =>   243 = 3^5
    log[2] i = 1 + log[3] 3^5  =>  log[b] a^z = z*log[b] a
    log[2] i = 1 + 5*log[3] 3   =>  log[b] b = 1 (base e logaritmando com mesmo valor sempre será 1)
    log[2] i = 1 + 5*1
    log[2] i = 6   => log[b] a = x => b = x^a
    i = 2^6

    i = 64

    S: { 6 }

                 

  • Em primeiro lugar calcule f(243) 

    f(x) = 1 + log3 x 

    f(243) = 1 + log3 243 

    Como, 

    243 = 3.3.3.3.3 = 3^5 

    f(243) = 1 + log3 3^5 

    f(243) = 1 + 5 log3 3 {Propriedade da potência de logaritmo} 

    f(243) = 1 + 5.1 { log3 3 = 1 pois, quando o logaritmando é igual a base o logaritmo vale 1} 

    f(243) = 1 + 5 

    f(243) = 6 

    Agora use a segunda equação, 

    g(x) = log2 x . Como g(i) = f(243) 

    g(i) = log2 i 

    g(i) = f(243) = 6 

    6 = log2 i 

    Pela definição de logaritmo, 

    Se 6 = log2 i i = 2^6 

    i = 2.2.2.2.2.2 

    i = 64=> alternativa (E)

  • Usando as propriedades dos logaritmos:

    f(x) = 1 + log3(x)

    g(x) = log2(x)

    g(i) = f(243) 

    Sabemos que f(243) = f(35) = 1 + log3(35) = 1 + 5log3(3) = 1 + 5 = 6

    Assim g(i) = log2(i) = f(35) = 6, logo:

    log2(i) = 6 → i = 26 = 64 anos


    Resposta: Alternativa E.
  • no final ele disse: f(243).

    logo, f(x) -> f(243)

    f(x) = 1 + log3(243)

    x=1+5

    x=6

    ________________________________________

    g(x) = log2x

    log2(x) = 6

    x=64 ∵2^6=64


ID
335215
Banca
CESPE / CEBRASPE
Órgão
CBM-ES
Ano
2011
Provas
Disciplina
Matemática
Assuntos

A soma dos logaritmos na base 10 de 2 números é 6, e o dobro de um desses logaritmos é 4. Com relação a esses números, julgue os itens a seguir.
O produto desses números é igual a 1 milhão.

Alternativas
Comentários
  • Sejam x e y os números procurados.
    logx + logy = 6
    log(x.y) = 6
    x.y = 106 = 1 milhão
    Correta a acertiva.
  • bom pessol lembrando que:   loga(B.C)  =  logaB  +  LogaC

    então   log10V  +  log10W  =  6
                 log10(V.W) = 10= 1000000
  • Chamei os números de a e b

    Log de a na base 10 + log de b na base 10 = 6

    log de (axb) na base 10 = 6

    axb = 10^6


ID
335218
Banca
CESPE / CEBRASPE
Órgão
CBM-ES
Ano
2011
Provas
Disciplina
Matemática
Assuntos

A soma dos logaritmos na base 10 de 2 números é 6, e o dobro de um desses logaritmos é 4. Com relação a esses números, julgue os itens a seguir.
A soma desses números é igual a 2.000.

Alternativas
Comentários
  • 2. log a = 4                                                                  
    log a^2 = 4
    10^4 = a^2
    a = 100

    log a + log b = 6
    log 100 + log b = 6
    2 + log b = 6
    log b = 4
    b = 10^4
    b = 10.000
     logo a + b = 10.100 (falsa questão)
  • LogX10+logY10=6

    2*Log10X=4

    Log10 X=4/2

    Log10 X=2
    X=10/\2
    x=100

    LogX10+logY10=6
    2+logY10 =6

    logY10 =6-2

    logY10 =4

    Y=10/\4
    Y=10000
    S=Y+X
    S=10100
  • Supomos que x e y sejam os números que estamos procurando, assim, de acordo com o enunciado:

    1) logx + logy = 6

    2) 2logx = 4 → logx = 2  (Aqui também poderíamos ter usado o logy)

    Assim (2) → (1):

    2 + logy = 6

    logy = 4 → y = 104 = 10.000

    Logo, logx = 2 → x = 102 = 100


    Assim, x + y = 10.000 + 100 = 10.100



    Resposta: Errado.
  • GAB E

    10^4+10^2=10100

  • Pessoal não tem oq fazer não ??? Elaborando questões assim !


ID
346681
Banca
CESPE / CEBRASPE
Órgão
PM-ES
Ano
2010
Provas
Disciplina
Matemática
Assuntos

Julgue os itens que se seguem, a respeito de operações com
logaritmos.

Se log5 b = 0,1,  em que b é um número positivo, então logb  25=0,01.

Alternativas
Comentários
  • log5 b=0,1 --> 5^0,1=b

    log b 25=0,01--> b^0,01=25

    Ou seja, 

    (5^0,1)^0,01=25 --> 5^0,001= 25. (Absurdo!)

    Pois, log5 25=2 

     

  • log5 b = 0,1
    5^0,1 = b
    b = 5^0,1 

    logb 25 = x (vamos ver se substituindo o b encontrado logo acima dá 0,01; como a questão diz)
    (5^0,1)^x = 25
    (5^0,1)^x = 5^2

    0,1x = 2

    x = 20 

    ERRADO, é igual a 20!
     

  • logb 25 = logb 5^2 = 2logb 5 = 2(log5 5 /log5 b) = 2 x 1/0,1 = 20

    Termo em destaque foi a aplicação da mudança de base do log

    GAB E


ID
346684
Banca
CESPE / CEBRASPE
Órgão
PM-ES
Ano
2010
Provas
Disciplina
Matemática
Assuntos

Julgue os itens que se seguem, a respeito de operações com
logaritmos.

Tomando 0,301  e  0,477  como os valores aproximados de log10  2   e  log10  3,  respectivamente, é correto inferir que log10  72 = 1,578.   

Alternativas
Comentários
  • 72 = 2^3 x 3^2

    log10 (2^3 x 3^2) = log10(2^3) + log10(3^2) = 3xlog10(2) + 2xlog10(3) = 3x0,301 + 2x0,477 = 0,903 + 0,954 = 1,857

    Resposta: Errada

  • ERRADO

    1,857

  • resultado de log 72 você ignora, já que vai tentar descobrir se ele é verdadeiro ou não.

    com isso, você faz: log 72 (base 10) = x

    vai encontrar 10^x = 72

    fatora 72 = 2^3 e 3^2

    questão deu resultado de log2 e log 3

    substitui e multiplica pelos expoentes (usando a propriedade)

    resultado será 1,857


ID
366784
Banca
NCE-UFRJ
Órgão
UFRJ
Ano
2009
Provas
Disciplina
Matemática
Assuntos

Nessa questão considere log 2≅0,301 e log 1,05≅0,021. Uma aplicação financeira cresce de acordo com uma Progressão Geométrica de razão 1,05 ao ano. Iniciando uma aplicação com 10 mil reais e não mais realizando operações de depósito e retirada, o número de anos que levará para que esse valor dobre está entre:

Alternativas
Comentários
  • PG ----> a1 = 10, an = 20, q = 1,05

    an = a1*q^(n - 1)

    20 = 10*1,05^(n - 1)

    2 = 1,05^(n - 1) ---- Aplicando log:

    log2 = log[1,05^(n - 1)

    log2 = (n - 1)*log1,05

    n - 1 = log2/log1,05

    n - 1 = 0,301/0,021

    n - 1 ~= 14,3

    n ~= 15,3 anos ----> Alternativa A

    fonte: http://pir2.forumeiros.com/t7345-log-em-matematica-financeira-2

  • GABARITO: alternativa a.


    log 2 ≅ 0,301 e log 1,05 ≅ 0,021
    q = 1,05

    a1 = 10.000

    an = 20.000

    para n > m, an = am x qn-m

    20.000 = 10.000 . 1,05^^(n - 1)

    2 = 1,05^^(n - 1)

    log 2 = log 1,05^^(n - 1)

    log 2 = (n - 1) . log 1,05

    0,301 = (n - 1) . 0,021

    n ≅ 15,333...


  • já que é uma aplicação em função do tempo da pra utilizar os juros compostos=> m=c.(1+i)^t

    20000=10000.(1+0.05)^t

    2=1,05^t

    log2=t.log1,05 , calcula e corre pro abraço

     

  • Uma aplicação de juros compostos.

    Aumentar 5% significa multiplicar por um 1,05. (É necessário ter o conhecimento básico de porcentagem).

    M= C. (1+i) ^ n

    20.000= 10.000. (1,05) ^ n (passa o 10 mil para o outro lado dividindo).

    2= 1,05 ^ n

    Log 2= Log 1,05 ^ n

    0,301= n . 0,021

    n= 14, 33...

    Gabarito Letra a.


ID
425920
Banca
UFBA
Órgão
UFBA
Ano
2009
Provas
Disciplina
Matemática
Assuntos

Aplicando-se um capital C por um prazo de t anos, em uma instituição bancária, o investidor obterá um montante M(t) = C (1,28)t .

Considerando-se log 2 = 0,30, pode-se afirmar que, para duplicar o capital aplicado, o prazo do investimento não poderá ser inferior a 36 meses.

Alternativas
Comentários
  • M = C (1,28)t

    No prazo t q ainda não sabemos, o capital duplica, ou seja, M passa a ser 2C

    2C = C(1,28)t   (divide por C, ou simplesmente cancela C)
    2 = (1,28)t

    Agora transforma em logarítimo

    log2 = log(1,28)t
    log2 = t(log1,28)
    log2 = t[log128 - log100]
    log2 = t[7.log2 - 2log10]
    0,30 = t[7.(0,30) - 2.(1)]
    0,30 = t[ 2,1 - 2]
    0,1t = 0,30
    t = 3 anos


    3 anos. 12 = 36 meses
  • um modo mais fácil:
    a taxa de juros anual é de 28%.

    para três anos, usando a fórmula, teremos (1,28)³ que é = 2.097.
    Logo com três ciclos, dobra-se o capital., com dois ciclos 1,28² da menos do que dois.
    logo, ducplica-se o capital a partir de tr6es períodos de cobrança.

ID
515815
Banca
Exército
Órgão
EsPCEx
Ano
2010
Provas
Disciplina
Matemática
Assuntos

O conjunto-solução da inequação xlogx(x+1)2 ≤ 4,  no conjunto dos números Reais, é

Alternativas
Comentários
  • Ao desenvolver a inequação chegamos ao resultado expresso na letra D.

    Porém, devemos lembrar da condição de existência da base do logaritmo (x>0 e x diferente de 1) e as condições de existência do logaritmando que, nesse caso, não adicionarão nenhuma restrição.

    Portanto, Gab A.

    Cuidar pra não cair mais nessas "pegadinhas".

  • Relembrando a propriedade dos logaritmos:

    Um número a elevado a um logaritmo de logaritmando b e base a é o próprio b.

    Portanto:

    (x+1)² <= 4

    Desenvolvendo:

    x² + 2x - 3 <= 0

    x1 = -3 e x2 = 1

    Relembrando a condição de existência dos logaritmos:

    O logaritmando deve ser maior que 0 e a base deve ser maior que 0 e diferente de 1, portanto x > 0 e x é diferente de 1.

    S = {x e R| 0 < x < 1}

    Alternativa A

    BRASIL!

  • Dá pra confudir pelo motivo de quer o qconcurso não colocou o x mais abaixo, ai da a entender que esse x é o meu logaritimando e não minha base

  • Haaa Qconcursos F*D*D* coloca a QUESTÃO DE FORMA CERTA, meu amigo a forma certa não tinha nada HAVER e ainda eles não colocam um GABARITO COMENTADO.

    https://www.youtube.com/watch?v=JnXEc5n0XBI

    Só errei por causa da plataforma é peso

    EDIT:REPORTEM O ERRO, PARA VER SE ELES CONCERTAM.


ID
521083
Banca
CESGRANRIO
Órgão
Petrobras
Ano
2005
Provas
Disciplina
Matemática
Assuntos

Quanto vale x, se log10 x3 - log10 x = 4?

Alternativas
Comentários
  • Essa questão utiliza a consequência da definição log de A na base A elevado a M é igual a M. Então, log de 100, 10², na base 10 elevado a 3 é igual a 6. E log de 100, 10² na base 10 é igual a 2. Como os resultados obtidos são 6 e 2, respectivamente, só é necessário, agora fazer a subtração que dá valor igual a 4. Obs.: Para chegar no resultado, no meu caso, precisei efetuar o teste com o valores das alternativas.

  • Log x³ - Log x = 4

     

    3 Log x - Log x = 4

     

    2 Log x = 4

     

    Log x = 4/2

    --- aplicanod a definição = 10² = x

     

    X = 100

     

     

     

     

  • log x^3-log x=4

    log x^3/x=4

    10^4=x^3/x

    10^4.x=x^3

    simplifica por x para virar uma equação do segundo grau...

    x^2=10^4

    x^2=10.000

    x=raiz quadrada de 10.000

    x=100

  • log x³- log x = 4, na base 10

    propriedade da subtração surgiu como uma solução do quociente entre os logaritmando

    log x³- log x= log x³/x

    log x ² , na base 10

    logx²=4

    10⁴=x²

    x= √10.000

    x= 100

    LETRA B


ID
545374
Banca
CESGRANRIO
Órgão
PETROQUÍMICA SUAPE
Ano
2011
Provas
Disciplina
Matemática
Assuntos

Um reservatório, completamente fechado e contendo ar, está conectado a uma bomba de vácuo. A bomba ligada, por um minuto, retira 10% da quantidade de ar desse reservatório. Quantos minutos, aproximadamente, a bomba deve ficar ligada para retirar 90% da quantidade de ar do reservatório?

Dado: log3 = 0,477

Alternativas
Comentários
  • Fórmula geral pela análise do problema: 
    P = Po * (0,9^t)  , logo: 
    0,1Po = Po * (0,9^t) 

    0,1 = 0,9^t

    log (0,1) = log (0,9^t)

    log (1/10) = t * log (9/10)

    - 1 = t * [ 2 log3 - 1 ]

    - 1 = t * [0,96 - 1] 

    t = 25 minutos.... O que faz a questão ser "má" é pedir aproximado (quando se chega a uma valor exato... e pior com 2 valores "aproximados" possíveis nas alternativas... d) 22 e e) 28...

  • Tharley, obrigado pela dica; todavia, você se equivocou no final. A resposta utilizando todos os algarismos significativos dados na questão é 21,7, aproximadamente 22. 

  • ok,aí depois que chegar ao resultado de n = 25 minutos aí faz-se a regra de tres assim

    25 minutos_____ 100% aí multiplica em cruz 25 x 90 = 100X x=22,5 minutos que é aproximadamente 22 minutos

    x _____ 90%

  • A taxa faz com que o ar diminua, portanto, 1 - 0,10 = 0,9. Tem que consumir 90%, então tem que partir do reservatório cheio (100% e chegar em 10%). Então, fica assim: 0,1 = 1.(0,9)^t

    log1 - log10 = log 1 + t[(log9) - (log10)]

    0 - 1 = 0 + t[2log3 - 1]

    -1= t(2.0,477 - 1)

    -1 = t(0,954 -1)

    -1 = -0,046t

    t= 1/0,046 = 21,74 min, ou seja, letra D.

  • Vídeo com a solução da questão

    profº Marcelo Leite

    https://www.youtube.com/watch?v=KV9A-sU9YlU

    minuto: 37:23


ID
551110
Banca
CESGRANRIO
Órgão
Petrobras
Ano
2011
Provas
Disciplina
Matemática
Assuntos

Dado  log 3(2) = 0,63, tem-se que  log6(24) é igual a

Alternativas
Comentários
  • log 3 (2) = 0,63 é o mesmo que 3 ^ 0,63 = 2 ^1

    log 6 (24) = x é o mesmo que 6 ^x = 24

    então:

    6 ^ x = 24 

    (3 * 2) ^ x = 3 * 2 ^ 3 

    substituindo:

    ( 3 * (3 ^ 0,63) ^ x = 3 * (3 ^ 0,63) ^ 3

    3^(1,63x) = 3^2,89 

    1,63x = 2,89

    x = 2,89/1,63

    x = 1,77 //

  • inutil explicação ... na prova nao saberemos quanto valera 3^0.63

  • Claro que saberemos Asael, a questão disse exatamente isso.

  • Log32 = 0,63, tem-se que Log624 é igual a:

    Log624 =

    Log63*8=

    Log63 + Log68=

    Log33/Log36 + Log323/Log36=

    Log33/Log32*3 + Log323/ Log32*3=

    Log33/(Log32 + Log33) + 3Log32/ (Log32+Log33)=

    SUBSTITUINDO

    1/(0,63 + 1) + 3X0,63/ (0,63+1)=

    1/1,63 + 1,89/ 1,63=

    1+1,89/1,63

    2,89/1,63

    1,77

    RESPOSTA:1,77

  • Solução limpa e agradável.

    Mudar de base o Log24 para base 3, onde já é dado o Log de 2 na base 3 que é 0,63.

    Log_3 (24) / Log_3 (6) = Log_3 [(2^3.3)/Log_3 (2.3)]

    = [3Log_3 (2) + Log_3 (3) / Log_3 (2) + Log_3 (3)]

    = [3.0,63 + 1] / [0,63 + 1]

    = 2,89/1,63 = 1,77

    Letra B


ID
568969
Banca
CESGRANRIO
Órgão
Petrobras
Ano
2010
Provas
Disciplina
Matemática
Assuntos

Uma população tem hoje P indivíduos e cresce a uma taxa constante de 25% ao ano. Sabendo-se que log102 = 0,30, estima-se que o número de indivíduos desta população daqui a vinte anos será

Alternativas
Comentários
  • (1,25)^20=n    (chamei de n o número que multiplicará a população daqui a vinte anos)

    20log1,25=logn

    20(log5/4)=logn

    20(log10-log2-log2-log2)=logn

    20(1-0,9)=logn

    logn=2 ==>100.

  • OBS da resposta do Willian:

    Log 5/4= Log 5- log 4

    • Sendo log 5= log (10/2)= log 10 - log 2= 1- 0,3 = 0,7
    • Log 4= 2*log 2 = 2*0,3 = 0,6

    Assim Log 5- log 4 = 0,7-0,6 = 0,1

  • É uma equação diferencial, resumidamente temos:

    dP/dt = kP e P=Po.exp(k.t) ; P = Po . exp(0,25.t) ; Se P=100Po ; ln(100) = 0,25t ; t = 20 anos


ID
583141
Banca
CESGRANRIO
Órgão
Petrobras
Ano
2006
Provas
Disciplina
Matemática
Assuntos

A taxa cobrada pela companhia A é o quádruplo da cobrada pela companhia B, para transporte da mesma carga. O plano da companhia A é reduzir em 15% sua taxa em cada mês, enquanto a companhia B aumenta 2% ao mês. Aproximando log2 = 0,301 e log3 = 0,477, em quantos meses o valor da companhia A será menor ou igual ao da companhia B?

Alternativas

ID
611497
Banca
COPEVE-UFAL
Órgão
Prefeitura de Penedo - AL
Ano
2010
Provas
Disciplina
Matemática
Assuntos

Assinale a opção incorreta.

Alternativas
Comentários

  • Comentando a errada (b)                                                                                 e é a base do logarítmo neperiano = ln
     
    e elevado a 2x =  e elevado a x ao quadrado ( expoente x não está aceitando no Word!)
     
    Fazendo uma mudança de variável: e elevado a x = t  fica  t²-3t +2 ≤ 0 dois valores inteiros
    em que a soma seja 3 e o produto 2 tantantantan  1e 2     parábens vc acertou!
     
    É uma parábola de concavidade voltado para baixo, como se fosse um U em que os valores
     
    negativos estão entre as raízes ou seja  1≤ t ≤ 2
     
    Mas t = e elevado a x 
     
     e elevado a x = 1   ou x = zero   (potência de expoente zero    é igual a unidade, base ≠ 0)
     
    e elevado a x =  2 ou   ln2  ( ln é o logaritmo na base e)
     
    resposta:  0≤ x ≤ ln2

    boa sorte

     
  • c) sim, pois são pssobilidades que não negativam

    d) 1dm³= 1 L = 1000cm³

    e) a100= 1+ (100-1)2

        a100= 1+ 198 ==> 199

  • Alguém poderia resolver a letra A ? :(

  • Testando cada alternativa até encontramos a incorreta:

    A) 1/3log7(x + 1) ≥ log7(2) → log7(x + 1)1/3 ≥ log7(2) → (x + 1)1/3 ≥ 2 → x + 1 ≥ 23 → x ≥ 7

    Alternativa correta.

    B) Lembrando que ln(ex) = x e fazendo a seguinte mudança de variável abaixo: i) y = ex

    ii) y² = (ex)² = e2x 
    Assim: e2x ≤ 3ex - 2 → y² ≤ 3y - 2 → y² - 3y + 2 ≤ 0 Resolvendo a equação de 2° grau utilizando a Fórmula de Bhaskara, encontraremos 2 raízes:y' = 1 e y'' = 2 Logo, de i):iii) 1 = ex → ln(ex) = ln(1) → x = 1 
    iv) 2 = ex → ln(ex) = ln(2) → x = ln(2)

    Logo, 1 ≤ x ≤ ln(2)

    Alternativa incorreta.


    C) Correto, pois se substituirmos qualquer valor do conjunto Dm(f) na função f(x), perceberemos que a raiz quadrada jamais irá ficar negativa, validando assim

    o item C.

    D) Alternativa correta, pois 1dm³= 1 L, logo 500dm³ = 500L.

    E) Alternativa correta, aplicando PA:

    a100 = 1+ (100 - 1)2 
    a100= 1+ 198 
    a100 = 199


    Resposta: Alternativa B. 

  • Eu fiz a letra A dessa forma...

     

    1/3 log 7(x+1) = log 7 2

    log 7(x+1) 1/3 = log 7 2

    (x+1) 1/3 = 2

    Elevando os dois lados a 3...

    X+1 = 2 3

    X +1 = 8

    Portanto X = 8-1 = 7


ID
620839
Banca
CONSULPLAN
Órgão
Correios
Ano
2008
Provas
Disciplina
Matemática
Assuntos

Sabendo que log10 2 ≅ 0,3 qual é o menor número natural que verifica a relação 2n > 104 ? ( ≅: aproximadamente)

Alternativas
Comentários
  • Temos a inequação:
    2n > 104
    Aplicando logaritmo em ambos os membros da inequação, esta não é alterada:
    log 2n > log 104 (lembrando que como a base do logaritmo é 10 e portanto maior que 1, então o sinal da desigualdade é mantido)
    Utilizando uma propriedade dos logaritmos:
    n . log 2 > 4 . log 10
    Conforme a questão, log 2 = 0,3, e devemos saber que log 10 = 1:
    n . 0,3 > 4 . 1
    n . 3/10 > 4
    n > 40/3
    Como 40/3 é aproximadamente 13,333...
    O menor número natural que verifica a relação dada é 14 (15 > 14 > 13,333...)
  • Por que "log 10 = 1" ?

    Alguém poderia explicar, por gentileza?
  • Respondendo a pergunta do Thiago.

    Por que "log 10 = 1"?

    dado: log b (x) = e , temos então: x = be 

    exemplo: log2 8 = e, temos então: 8 = 2e  -->  23=2e --> e = 3. portanto log2 8 = 3

    portanto temos que: log 10 = log10 10  --> 10 = 10e --> e = 1

    log 10 = 1

  • Obrigado Scheila.

    Compreendido agora.
  • De acordo com o enunciado, log10(2) ≈ 0,3. Aplicando as propriedades:

    2n > 104

    log10(2n) > log10(104)

    n.log10(2) > 4.log10(10)

    0,3n > 4   (Lembrando que log10(10) = 1)

    n > 4/0,3

    n > 13,33

    Assim, como n tem que ser maior que 13,33, o menor número que satisfaz n será o 14.
     

    Resposta: Alternativa D.
  • log 2^n>10^4

    n.0,3>4.1

    n>4/0,3

    n>13,33

  • OBS:o N tem que ser MAIOR que 4

    log2^n>log10^4

    n.log2>4.log10

    n.0,3>4.1

    n>4/0,3

    n>14 aproximadamente

  • Fiz várias vezes e até hoje não entendi.

    Se o resultado é 13,333, e no enunciado ele dizer "o menor número natural", o gabarito deveria ser 13 e não 14.

    Se o examinador dissesse: " o maior número natural", então seria o número 14, pois seria o próximo depois do número 13.


ID
635455
Banca
CEPERJ
Órgão
SEDUC-RJ
Ano
2011
Provas
Disciplina
Matemática
Assuntos

João tem uma fazenda de gado, e a quantidade de animais cresce regularmente 20% a cada ano. Certo dia, João diz: “se todas as condições continuarem as mesmas, daqui a n anos minha boiada será 10 vezes maior que a de hoje”. O menor valor inteiro de n que torna essa afirmação verdadeira é:
Obs: dado log12 = 1,08

Alternativas
Comentários
  • Termo geral PG

    an = a1.qn -1
    10x = x .(1,2)n -1  cancela x
    10 = (1,2)n -1
    10 = (1,2)n
               1,2

    1,2(10) = (1,2)n
    12 = (1,2)n

    Transforma em logarítmo

    log12 = log(1,2)n
    log12 = log(12/10)n
    log12 = n[ log12 - log10]
    1,08 = n[ 1,08 - 1]
    1,08 = n [ 0,08]
    0,08n = 1,08
    n = 13,5

    menor valor inteiro: 13
  • 10 = 1 * 1,2^n
    10 = 1,2^n
    n = log(1,2) 10 . . . . . . . . . . (log de 10 na base 1,2)

    Transformação para logaritmo decimal:
    n = log 10 / log 1,2
    n = log 10 / log (12/10)
    n = log 10 / (log 12 - log 10)
    n = 1 / (1,08 - 1)
    n = 1 / 0,08
    n = 100/8 = 25/2

    n = 12,5 anos

    Ou, arredondando:

    n = 13 anos
  • an=a1.qn-1

    10=1 . 1,2n -1

    10=1,2n-1

    Passando para logaritmo:

    Log10=log1,2n-1

    Log10 = (n-1).log1,2

    Log10=(n-1).log12/10

    Log10=(n-1).[log12-log10]

    1=(n-1).[1,08-1]

    1=(n-1).0,08

    1=0,08n-0,08

    1+0,08=0,08n

    n=1,08/0,08

    n=13,5

    menor valor inteiro = 13


  •  João tem uma fazenda de gado, e a quantidade de animais cresce regularmente 20%(= 100% + 20% = 120% = 1,2) a cada ano. Assim, temos uma PG, cuja razão será q = 1,2:


                                                                         an = a1*q(n-1) = a1*1,2(n-1)


    Daqui a n anos, a boiada será 10 vezes maior que a de hoje. Logo tomando a1 como a quantidade de bois que João tem atualmente: an = 10a1 assim:


    10a1 = a1*1,2(n-1)

    10 = 1,2(n-1)

    Aplicando log na base 10 em ambos os lados e lembrando que log(10) = 1:


    log(10) = log(1,2(n-1))

    1 = (n - 1)*log(1,2)

    1 = (n - 1)*log(12/10)


    1 = (n - 1)*[log(12) - log10]


    1 = (n - 1)*[log(12) - 1]


    1 = (n - 1)*[1,08 - 1]


    1 = (n - 1)*0,08


    0,08n = 1 + 0,08


    n = 1,08/0,08


    n = 13,5 ≈ 13 (Menor valor inteiro)



    Resposta: Alternativa B.

  • Eu discordo, para que o menor valor inteiro de n que torna a boida 10 vezes maior deveria ser o 15.

    Em apenas 13 anos ainda não será 10 vezes maior, por causa que levará 13,5 anos para chegar ao que se afirma. ¬¬

  • A maneira correta de resolver essa questão é por juros compostos e não PG. Pois a cada ano a boiada cresce 20%. M = C (1+i)^t B = Bois que é seu capital, Seu montante é o que vc quer ter 10B, logo: 10B = B * (1,2)^t   -> 10 = 1,2^t. Comentário da wanessa explica como resolver.
    É só lembrar que se eu tinha 100 bois no primeiro ano no segundo eu terei 120 e agora serão 20% dos 120 bois. 
    Por pg o resultado é 13,5 logo o menor valor inteiro seria 14
    Por Juros compostos o resultado é 12,5 logo o menor valor inteiro é 13 -> Resposta do gabarito.

  • Galera, se fizer por PG dá Merda, fiz por juros compostos e deu 12,5, arredondando pra cima, 13 anos.
    por P.G estava dando 13,5, logo o menos inteiro possível deveria ser 14.
    Ou seja, na P.G. observei que a galera olha só para o enésimo termo da PG, quando na verdade deveria fazer por soma dos termos da P.G, daí daria certinho, só que essa soma dos termos da P.G é justamente a equação simplificada de juros compostos.

  • Resposta!

    https://drive.google.com/open?id=1LEVfS7LRoKnIpfy-RYX_e9a7LQUTDsQ2

  • E possível chegar ao resultado pela formula dos juros compostos

    m=c(1+i)^t

    10b=b.(1+0,2)^t

    10b=b.1,2^t

    1,2^t=10b/b

    1,2^t=10

    log1,2^t=log10

    t.log1,2=log10

    t=log10/log1,2

    t=1/log12/log10

    t=1/log12-log10

    t=1/1,08-1

    t=1/0,08

    t=12,5

    A questão pede o menor valor INTEIRO de N,portanto a resposta e !3.


ID
655126
Banca
VUNESP
Órgão
UNIFESP
Ano
2007
Provas
Disciplina
Matemática
Assuntos

Uma das raízes da equação 22x – 8.2x + 12 = 0 é x = 1.

A outra raiz é

Alternativas
Comentários
  • Ola´
     
    Fazendo uma mudança de variável ; 2 elevado a x vamos chamar de t
    Então 2 elevado a dois x  é igual a dois elevado a x ao quadrado.
     
    Estou fazendo assim pq no Word não está aceitando expoente  x.
     
    Fica: t² - 8t + 12 = 0  em vez de Báskhara, dá para se achar algumas raízes de equação do 2º grau   facilmente pela relação existente entre coeficientes e raízes: quais são as duas raízes em que a soma é 8 e o produto 12. Resposta: 2 e 6 . Muito bem!
     
    2 elevado a x = 2 , x = 1 (resposta dada)
     
    2 elevado a x = 6. aqui vamos usar logaritmos, pois a resposta  do expoente x não é exata.
     
     log2 elevado a x = log 6→  xlog2 = log6  (propriedade das potências de logaritmos)
     
    xlog2 = log2*3  → xlog2 = log2 + log3 (produto de um logaritmo é igual a soma de logaritmos dos fatores)
     
    x = log2/log2 + log3/log2  =  1 + log 3/2 ( divisão de logaritmos de mesma base é igual ao logaritmo do quociente dos logaritmandos)
  • 2^x = 6==> log de 6 na base 2 , eu ja passei pra base 10, ficou log6/log2==> log(3.2) / log 2 ===> log 3 + log 2 - log 2==> deu como resultado log 3 , no que eu errei?

  • Quando desenvolve a equação é encontrado do 2^x = 2(raiz já mencionada pela questão) e 2^x=6.

    Transformando 2^x=6 em log, ficamos com log6/log2 -> log(3.2)/log2 -> (log3+log2)/log2 ->

    log3/log2 + log2/log2 -> log3/log2 + 1


ID
673153
Banca
UFMG
Órgão
UFMG
Ano
2008
Provas
Disciplina
Matemática
Assuntos

Numa calculadora científca, ao se digitar um número positivo qualquer e, em seguida, se apertar a tecla log, aparece, no visor, o logaritmo decimal do número inicialmente digitado.

Digita-se o número 10.000 nessa calculadora e, logo após, aperta-se, N vezes, a tecla log, até aparecer um número negativo no visor.

Então, é CORRETO afrmar que o número N é igual a

Alternativas
Comentários
  • Gabarito B.

    10000 = 10^4

    Primeiro aperto. Log10^4 = 4 

    Segundo Aperto. 10^0 > 4 > 10^1 (propriedades de potencia), logo Log10^0 > log4 > Log10^1 = 0 > Log4 > 1. Assim, podemos presumir que Log4 esteja entre 0 e 1, ou seja, Log4 = 0,X

    Terceiro Aperto. Log (0,X) > 0.

    Numero de apertos = 3


ID
682741
Banca
VUNESP
Órgão
UNIFESP
Ano
2006
Provas
Disciplina
Matemática
Assuntos

A relação P(t) = P0(1 + r)t onde r > 0 é constante, representa uma quantidade P que cresce exponencialmente em função do tempo t > 0. P0 é a quantidade inicial e r é a taxa de crescimento num dado período de tempo. Neste caso, o tempo de dobra da quantidade é o período de tempo necessário para ela dobrar. O tempo de dobra T pode ser calculado pela fórmula

Alternativas
Comentários
  • Resolvendo:

    P (t) = 2P0 ↔ P0 (1 + r)t = 2P0 ↔ t = log(1+r)2

    Letra A.



ID
691156
Banca
UDESC
Órgão
UDESC
Ano
2010
Provas
Disciplina
Matemática
Assuntos

Seja S a soma dos seis primeiros termos de uma progressão geométrica de razão igual a 1/2 . Se log S  = 2 log 2 + log7, então o primeiro termo desta progressão é igual a:

Alternativas

ID
691420
Banca
UDESC
Órgão
UDESC
Ano
2011
Provas
Disciplina
Matemática
Assuntos

A escala Richter foi criada em 1935 para quantificar a intensidade de energia liberada por um abalo sísmico, utilizando-se um aparelho chamado sismógrafo. Os pontos nesta escala são um logaritmo na base 10 da quantidade de energia liberada. O terremoto ocorrido no Nordeste do Japão em 11 de março de 2011 atingiu 9 pontos nesta escala, enquanto o terremoto mais intenso registrado na história, ocorrido no Chile em 1960, atingiu 9,5 pontos nesta escala.
É correto afirmar que a quantidade de energia liberada no sismo do Chile foi:

Alternativas

ID
691432
Banca
UDESC
Órgão
UDESC
Ano
2011
Provas
Disciplina
Matemática
Assuntos

Sejam a, b e c números reais positivos tais que

log2 a+ log1/4 b- log1/2 c = 3

Então b é igual a:

Alternativas

ID
701140
Banca
CESGRANRIO
Órgão
Petrobras
Ano
2012
Provas
Disciplina
Matemática
Assuntos

Se y=log81 (1/27) e x ∈ IR+ são tais que xy = 8 , então x é igual a

Alternativas
Comentários
  • Resolvendo o log acharemos -3/4.

     

    Substituindo em x^y = 8  ficará: X^-3/4=8

     

    Para tirar o sinal negativo, passando para fração: (1/X)^3/4 = 8

     

    Passando o expoente para o outro lado irá inverte-lo : 1/X= 8^4/3 ----> X= Raiz cúbica de 8^4 ----> X = 16

     

    Substituindo o X na equação 1/X^3/4 = 8 ----> 1/16 = 8 ----> Resposta 1/16.  A

  • como achou este log ?

  • y=log na base 81 de 1/27.

    81^z=1/27 --> 3⁴z = 1/3³ --> 3⁴z=3³(exopoente negativo, aqui ficou ruim demostrar na fórmula, mas vc tem que entender que subindo o três, o expoente fica negativo).

    Então temos, 4z = -3 --> z=-3/4 ou seja, log na base 81 de 1/27 é igual a -3/4

    Espero ter ajudado.

  • log (81) 1/27

    log (3^4) 3^-3

    -3/4 log(3) 3 = -3/4 se x^y = 8 então x^-3/4 = 8

    log(x) 8 = -3/4

    log(2) 2³ / log(2) x = -3/4

    3 / log(2) x =-3/4 multiplica em cruz

    12 = -3 log(2) x

    log (2) x = -4

    x = 2^-4 para tirar o sinal negativo do expoente invertemos

    x = (1/2)^4 logo x=1/16 

    RESPOSTA LETRA A

     

  • log 3^-3 = 3^4.y

    4y = -3

    y = -3/4

    x ^ -3/4 = 2^3

    Multiplica-se ambos expoentes por -4/3 para que X fique elevado a potência 1. Desta forma, obtemos:

    x ^ -3/4 . -4/3 = 2^3. -4 /3

    x^1=2^-4 

    x=1/16

    Gabarito letra A

  • Ronald, q prpriedade foi essa que passou o expoente para o outro lado?? 

    Cheguei na parte 1/X ^ 3/4 = 8...... não saio daqui.


ID
701992
Banca
CESGRANRIO
Órgão
Petrobras
Ano
2012
Provas
Disciplina
Matemática
Assuntos

Se  y = log81[1/27] e x ∈ IR+  são tais que xy = 8 , então x é igual a

Alternativas
Comentários
  • y=log81[3^-3]

    81^y=[3^-3]

    3^4y=3^-3

    4y=-3

    y= -3/4

    Substitui-se o valor do y na equação X^Y=8

    x^[-3/4] = 2^3 ---- multiplica-se em ambos os lados por (-4/3), para que se simplifique o expoente de X

    x^([-3/4]* [-4/3]) = 2^(3*[-4/3])

    x = 2^[-12/3]

    x = 1/[2^4]

    x = 1/16

    gabarito A


ID
702127
Banca
CESGRANRIO
Órgão
Petrobras
Ano
2012
Provas
Disciplina
Matemática
Assuntos

Se y = log81 ( 1/27) e x ∈  IR são tais que xy = 8 , então x é igual a

Alternativas

ID
702262
Banca
CESGRANRIO
Órgão
Petrobras
Ano
2012
Provas
Disciplina
Matemática
Assuntos

Se y=log81(1/27) e  x ∈ IR+ são tais que xy = 8 , então x é igual a

Alternativas
Comentários
  • 81^y=1/27

    3^4y = 3^-3

    4y = -3

    y = -3/4


    Substituindo x^y=8


    x ^ -3/4 = 8

    x^-3/4 = 2^3


    Multiplica-se ambos expoentes por -4/3 para que X fique elevado a potência 1. Desta forma, obtemos:


    x ^ -3/4.-4/3 = 2^ 3.-4/3


    x ^12/12 = 2^-12/3


    x^1 = 2^-4 


    x=1/16

  • Olá Nancy, boa noite!

    Poderia resolver novamente porém com mais detalhes nos cálculos?

  • Nancy, poderia explicar pq multiplicar o expoente por -4/3??? 

  • Em negrito é a base do logarítimo 

    y=log81(1/27)=> log81 3^-3=>

    Y=log3^4 3^-3=>

    Aplicando a propriedade dos expoentes dos lagritimandos:

    Y= -3/4 log3 3 (log3 3=1)

    Y= -3/4 *1

    Y=-3/4

    ###################################

    X^y=8

    X^-3/4=2^3

    Para excluirmos o exponte de x mutiplicamos os dois por ^4/3

    X^-3/4=2^3 (^4/3)

    x^-1=2^4

    Para excluirmos o exponte de x mutiplicamos os dois por ^-1

    x=2^-4

    x=1/16


ID
705247
Banca
CESGRANRIO
Órgão
Petrobras
Ano
2012
Provas
Disciplina
Matemática
Assuntos

Se y = log81 (127) e x ∈ IR+    são tais que xy = 8 , então x é igual a

Alternativas
Comentários
  • Imagem 022.jpg
    Log81(1/27)=y -> 81^X=1/3^3

    3^4x=3^-3

    x=´-3/4

    xy=8
    -3/4*x=8
    x=-6
  • O enunciado da questão está errado. Segue a correta:



  • Log81(1/27) = Y -> 81^Y = 1/3^3
    3^4Y = 3^-3
    Y = -3/4
    XY= 8
    X-3/4=8
    1/X4/3 =8
    1=8.X4/3( ficará raiz cúbica de 8 elevado a 4)
    1=16X
    1/16=X
  • Há um erro nas propriedades aplicadas no comentário anterior por leilane rodrigues :Log81(1/27) = Y -> 81^Y = 1/3^3

    3^4Y = 3^-3

    Y = -3/4

    XY= 8

    X-3/4=8

    1/X4/3 =8    (quando deveria ser) ====>  1/X3/4=8

    (pois a-n = 1/an ), não (a-n = 1/a1/n ). Logo o comentário anterior não está correto.

    1=8.X4/3( ficará raiz cúbica de 8 elevado a 4)

    1=16X

    1/16=X
  •  

    na verdade a amiga do comentario acima se equivocou ao invez de ( 1/x )4/3  o certo seria ---

    (1 / x )3/4 = 8
    1= 8 * x 3/4 
    1/8 = x 3/4
    aplica raiz cúbica em (1/8)4 = (1/23)4= (1/212
     raiz cúbica de (1/212) = (1/212)1/3 = (1/24) =  1/16

  • y = log (81) (1/27) 

    y = -3log(81)(3) 

    y = -3*1/4 
    y = -3/4 

    #################### 

    x^(-3/4) = 8 

    Elevando os dois termos à quarta potência: 

    x^-3 = 8^4 

    1/x^3 = 8^4 

    Agora raiz cubica dos dois termos: 

    1/x = 8^4/3 

    Raíz cúbica de 8 = 2 

    1/x = 2^4 
    1/x = 16 

    x = 1/16 <=== RESPOSTA

  • Resolvendo: 

                                             
    Letra A

  • Temos Log de (1/27) na base 81, isso significa que o 81 elevado a algum número (que aqui chamaremos de Z), resultará em 1/27. Logo temos que:

    81^z = 1/27

    Decompomos a expressão e chegamos em:

    3^(4z) = 3^(-3)

    Lembrando que 81 é a mesma coisa que 3 elevado a 4, que multiplica o z já existente no expoente. Por outro lado temos que 3x3x3=27, para jogarmos o 27 que está em baixo como numerador, elevamos a -1. Nesse caso como temos 3^3, ficará 
    3^ (-1).(3) = 3^-3.

    Se temos a mesma base agora resolvemos a expressão:

    4z=-3
    z=-3/4

    Tendo o valor de Z em mãos, agora substituímos na expressão que contém o X. Nosso Z será o Y requerido.

    X ^ y = 8
    x ^-3/4=8

    Elevando todos os termos à quarta potência, iremos eliminar a divisão por quatro do expoente -3/4. Logo teremos:

    x ^ (-3/4)*4 = 8 ^ 4
    x ^ (-3) = 8 ^ 4

    Se temos x elevado a um expoente negativo, para deixarmos positivo retornamos o X à fração:

    1/x ^ 3 = 8 ^ 4

    Agora colocamos todos os termos em raiz cúbica, a fim de eliminarmos o expoente 3 que está em x:

    raiz cúbica de 1/x ^3 = 1/x
    raiz cúbica de 8 ^ 4 = 2 ^ 4

    Consequentemente teremos que:

    1/x=2 ^ 4
    1/x = 16
    x= 1/16

    alternativa A.

  • Sei que tem um monte de respostas aqui, então vou colocar o meu passo-a-passo também =)

    y = Log (1/27) na base 81, então se tem divisão no log, joga na propriedade de divisão => Log 1/27 => Log 1 - Log 27.
    y = Log 1 - Log 27 (todos na base 81), lembrando que todo log 1 em qualquer base é 0.
    y = 0 - Log 27 (base 81), agora joga na propriedade de igualar base, mas como dá para ver que tudo está relacionado com o número 3, coloca tudo na base 3.
    y = - log 27 (base 3) / log 81 (base 3), agora é só calcular normalmente. Para calcular isso é só pensar:  bem, tenho a base 3, então tenho que saber que exponencial preciso usar para chegar em 27, desse jeito ó:
    3^x=27
    Opa, 3^3 = 27, então
    Log 27 (na base 3) = 3.
    Mesma ideia vale para o outro,
    3^x=81
    3^4 = 81, então
    Log 81 (na base 3) = 4
    Daí é só substituir.
    y = - log 27 (base 3) / log 81 (base 3)
    y = - 3 / 4

    Beleza, agora sabendo que x^y=8, então

    x^-3/4=8

    Agora é só lembrar dessa propriedade abaixo:
    x^2 = 4, manda o 2 para o outro lado como raiz.
    x = Raiz Quadrada de 4
    x = 2

    Quando você manda para o outro lado, você só está invertendo o bicho, transformando ele em exponencial 1/2, que nada mais é que raiz quadrada mesmo.

    x^2=4, manda o 2 para o outro lado como raiz.
    x=4^1/2, que é a mesma coisa que raiz quadrada.
    x= Raiz Quadrada de 4
    x = 2

    Então é só usar a mesma lógica de jogar a exponencial para o outro lado, deixando ele invertido
    x^-3/4 = 8
    x = 8^-4/3, como não queremos fazer uma raiz cúbica com exponencial negativa, é só lembrar que 8 = 2^3,
    x = (2^3)^-4/3, propriedade de exponencial em exponencial, é só multiplicar as exponenciais.
    x = 2^3*-4/3
    x = 2^-12/3
    x = 2^-4, para tirar esse negativo é só colocar 1 em cima de tudo.
    x = 1/2^4
    x = 1/16.

    Como curiosidade mesmo:

    Lembrando que todos os passos a seguir é só para comprovar como a exponencial chegou invertida do outro lado no primeiro passo:

    Tínhamos
    x^-3/4=8
    Então Logaritmiza tudo!
    log x^-3/4 = log 8, propriedade de exponencial, desce o bicho já multiplicando.
    -3/4 * log x = log 8, tá multiplicando, passa para o outro lado dividindo.
    log x = log 8 / -3/4, agora que é o negócio, todo número já é dividido por 1, ou seja, (log 8)/1 / -3/4, então, é só continuar com a conta, conservando a primeira e multiplicando pelo inverso da segunda:
    log x = log 8 * -4/3, esse é o inverso da propriedade do exponencial, só colocar o que está multiplicando como exponencial.
    log x = log 8^-4/3, quando tem logs de mesma base, só cortar eles.
    x = 8^-4/3.
    Pronto =)

    Sei que não é simples, mas é só saber as propriedades que fica tranquilo.

  • y=log81^1/27

    y=log 3^4 ^3^-3

    y=-3/4

    x^y=8

    x^-3/4=8

    1/x^3=8^4

    1/x=8^4/3

    x= raíz cúbica de 8 = 2

    1/x = 2^4

    1/x=16

    x=1/16


ID
713659
Banca
CESGRANRIO
Órgão
Petrobras
Ano
2011
Provas
Disciplina
Matemática
Assuntos

Qual é o valor mínimo assumido pela função f : ℜ*+ →ℜ , definida por f(x) = x2 In(x)?

Obs.:
e é o número de Euler
ln(x) é o logaritmo natural de x

Alternativas
Comentários
  • O valor mínimo de f(x) pode ser encontrado fazendo-se f'(x) = 0

    f'(x) = 2x*lnx + x =0

    x(2lnx + 1) = 0

    x = e^-1/2

    Substituindo em f(x):

    f(e^-1/2) = (e^-1/2)^2*ln(e^-1/2) = (e^-1)*(-1/2) = -1/2e

     

    Gabarito: D

     

    Bom estudo a todos!

     

     


ID
713872
Banca
UECE-CEV
Órgão
UECE
Ano
2011
Provas
Disciplina
Matemática
Assuntos

Se os números reais positivos m, n, e p formam, nesta ordem, uma progressão geométrica, então a soma log m + log n + log p é igual a

Alternativas

ID
716701
Banca
UECE-CEV
Órgão
UECE
Ano
2010
Provas
Disciplina
Matemática
Assuntos

Se x, y, z e w são as raízes da equação x4 + 2x2 + 1 = 0, então

log2|x| + log2|y|+ log2|z|+ log2|w| é igual a

Alternativas

ID
733519
Banca
Exército
Órgão
EsPCEx
Ano
2011
Provas
Disciplina
Matemática
Assuntos

Se x é um número real positivo, então a sequência (log3x, log33x , log39x) é

Alternativas
Comentários
  • 1 = Log 3 na base 3

    Log X na base 3 + Log 3 na base 3 = Log 3X na base 3

    Log 3X na base 3 +Log 3 na base 3  = Log 9x na base 3

    Logo, uma P.A com razão 1.

  • (log x, log 3x, log 9x) = (log x, log 3 + log x, log 3² + log x) = (log x, 1+logx, 2+logx) PA de razão 1.

    LEMBRANDO QUE OS LOGS ESTÃO TODOS NA BASE 3.

     


ID
846454
Banca
CESGRANRIO
Órgão
LIQUIGÁS
Ano
2012
Provas
Disciplina
Matemática
Assuntos

Qual é o produto das raízes da equação
[log(x)] 2 - log(x2 ) - 3 = 0 ?

Alternativas
Comentários
  • [Log (x) ]^2 - Log (x^2) - 3 = 0
     
     
     
    Substitua Log (x) = y
     
     
    y^2 - 2y -3 = 0
     
    raizes:
     
    y = 3
     
    y"= -1
     
     
    Substitua y = Log (x)
     
     
    Log X = 3 :. x = 10^3 :. x =1000
     
    Log x" = -1 :. x" = 10^-1 :. x=1/10
     
     
    Produto das Raizés:
     
    x * x" = 100
  • Alguém pode resolver essa? Tô precisado de ajuda...

  • log x = y

    y² - 2y - 3 = 0

    y = 3 e y = -1

    log x = 3 => x = 10³

    log x= -1 => x = 10^-1

    10³.10^-1=10²=100

  • Para resolvermos este problema, devemos lembrar das propriedades de logaritmo e como se resolve uma equação logarítmica. Vejamos:

    log(x2) pode ser escrito como 2.log(x), utilizando a propriedade logaritmo da potência (verifique!).

    [log(x)]2 – log(x2) – 3 = 0 , então [log(x)]2 – 2.log(x) – 3 = 0.

    Agora, vamos fazer uma substituição para nos ajudar na resolução, vamos fazer

    log(x) = y.

    [log(x)]2 – 2.log(x) – 3 = 0.

    y2 – 2.y – 3 = 0

    Resolvendo a equação do 2° grau acima:

    Mas, a equação original se encontra na incógnita x, então vamos “voltar”.

    Produto das raízes = 0,1.1000 = 100.

  • Não estou acostumado com essas questões.  Mas vamos em frente. 

  • Complicado essa questão. Muitas substituições 

  • Eu entendi que ele queria o produto das raízes da equação que são 3 e -1. Não o produto dos logs.

  • Qual é o produto das raízes da equação [log(x)]^2 - log(x^2 ) - 3 = 0 ?

    As raízes, para quem não sabe, é ou são os valores de x para quais a equação é igual a zero.

    Ele quer o produto das raízes dessa equação.

    Vamos encontrá-las então.

    Tomar cuidado com o sinal de menos ali no meio.

    [Log (x). Log (x)] - 2Log (x) - 3 = 0 (vamos usar a substituição de variável para encontrar o valor de Log (x), e não as raízes da equação.

    Dessa forma, visto q é possível nesse caso: Log (x) = y.

    Portanto, y^2 - 2y - 3 = 0

    Vamos fazer por Soma e Produto, visto que o coeficiente de a=1 (1y^2)

    Logo, S= 2, P = -3, logo = (3, -1), ou seja, 3-1 = 2, e 3*-1 = -3

    Essas não são as raízes da equação. São os valores de Log (x).

    Agora, vamos achar as raízes, ou seja, os valores de x.

    Primeira raíz,

    Utilizar a definição de Log.

    Log x1 = 3

    10^3 = x1

    Segunda raíz,

    Utilizar a definição de Log.

    Log x2 = -1

    10^-1 = x2

    Verificar a CE.

    x>0, então ambas as raízes atendem, ou seja, 10^3>0 e 10^-1>0

    Agora, basta calcular o produto delas,

    Portanto,

    Produto

    10^3 = 1000

    10^-1 = 1/10

    1000 x 1/10 = 100

    Letra D

  • Se substituir x por 1000 teremos log 1000 que é 3. Com isso teríamos 3²-2*3-3, o resultado disso é 0. Joguei essa equação também em solvers de equação e também cheguei a esse mesmo gabarito (letra E)


ID
884830
Banca
FUNCAB
Órgão
CBM-RO
Ano
2009
Provas
Disciplina
Matemática
Assuntos

Considerando que log 2 0,301 e que log 3 0,477, podemos dizer que a solução da equação exponencial 5x 48 000 vale, aproximadamente:

Alternativas
Comentários
  • Alguém me explica essa questão, por favor?!...

    Agradeço muito.

  • modelo: [ 5eX = 5 elevado a X ]

    5eX=48000, portanto, log48000(base5)

    Fazendo mudança de base:

    log48000(base10)/log5(base10) = log2e4x3x10e3/ log10/2

    Usando de propriedades dos logaritimos:

    log2e4 + log3 + log10e3 / log10 - log2

    4log2 + log3 + 3.log10 / log10 - log2

    4(0,301) + 0,477 + 3.1 / 1 - 0,301 = 1,204+0,477+3 / 0,699 = 4,681/0,699 = 6,697

    OBS: log2 e log3 foram dados na questão


ID
888526
Banca
VUNESP
Órgão
UNESP
Ano
2012
Provas
Disciplina
Matemática
Assuntos

Todo número inteiro positivo n pode ser escrito em sua notação científica como sendo n = k ·10x , em que k &isin; R*, 1 &le; k < 10 e x &isin; Z. Além disso, o número de algarismos de n é dado por (x + 1).

Sabendo que log 2 &cong; 0,30, o número de algarismos de 257 é

Alternativas
Comentários
  • n = 2^57
    logn = log(2^57)
    logn = 57.log2
    logn = 57(0,3) = 17,1
    n =10^(17,1)

    10¹ tem 2 algarismos
    10² tem 3 algarismos
    10³ tem 4 algarismos.Ou seja,o nº de algarismos á sempre 1 a mais do que a potência de 10.
    10^(17,1) tem, portanto, 18 algarismos.
    AlternativaC
    resposta segundo:

    http://br.answers.yahoo.com/question/index?qid=20130622071927AAqHdaw
  • Questão: "Todo número inteiro positivo n pode ser escrito como n = k * 10x, em que K pertence a R (números reais), 1 < K < 10; e x pertence a Z (números inteiros). Além disso, o número de algarismos de n é dado por (x+1). Sabendo que log 2 ~ 0,30, o número de algarismos de 257 é:”
     
    Resolução:
    Seja n = 257.
    Temos: logn = log257
    ∴logn = 57 ⋅log2

    Usando log2 = 0,3, temos:
    logn = 57 * 0,3 
    ∴logn = 17,1
    ∴ n = 1017,1 = 100,1 * 1017
    Note-se que, como 100= 1 e 101= 10, temos 1 < 10 0,1 < 10. Assim, representando n na forma k * 10 x , temos
    k = 10 0,1 e
    x = 17.
    Do enunciado, a quantidade de algarismos de n é (x + 1), ou seja, 18.
    Resposta: C


    Fonte: angloresolve.cursoanglo.com.br
    Provas resolvidas, unesp 2012, 1 fase, questão 90
    file:///C:/Users/DeskTop/Downloads/Prova_909_AR.pdf
    file:///C:/Users/DeskTop/Downloads/Prova_909_AR.pdf
  • De acordo com o enunciado temos que:

                                        n = 257 →  log(n) = log(257) =  57log2 = 57 (0,3) = 17,1

    Assim, 101 tem 2 algarismos, 102 tem 3 algarismos e 103 tem 4 algarismos. Ou seja, o nº de algarismos a sempre 1 amais do que a potência de 10. Logo 1017.1 tem, portanto 18 algarismos


    Letra C


ID
976420
Banca
Exército
Órgão
EsSA
Ano
2012
Provas
Disciplina
Matemática
Assuntos

Se log2 3 = a log2 5 = b ,então o valor de log 0,5 75 é:

Alternativas
Comentários
  • O enunciado está confuso, mas dá pra ler assim:
    log de 3 na base 2 é igual a A; e
    log de 5 na base 2 é igual a B.

    75 = 3.5²

    0,5 = 1/2 = 2 elevado na -1

    o expoente do logaritmando multiplica a expressão

    log5^2 = 2.b

    o expoente da base divide a expressão, então nesse caso específico é só inverter o sinal

    -(a + 2b) = -a -2b

  • Log0,5 75

    Log1/2 3 + Log1/2 5²

    Aqui o ponto chave : A conversão (1/A = a^-¹)

    Log(-2) 3 + 2Log(-2) 5

    - a - 2b

    Letra E

    APMBB

    https://uploaddeimagens.com.br/imagens/UStp6Zo

  • Hot question, baby

  • Vou tentar responder "fora da equação" pra tentar ser bem explicativo.

    1º : Fatoramos o 75 e achamos 3.5²

    2º: Simplificamos a base 0,5 como 5/10 e depois como 1/2

    3º: Faça o inverso de 1/2 que é igual a 2-¹

    4º: Aplica a propriedade que diz que o expoente da base multiplica seu inverso pelo logaritmo

    assim temos até aqui: 1/-1 (log 3.5² (base 2))

    vamos aplicar mais uma propriedade que diz que a multiplicação dos logaritmandos é a soma dos seus logaritmos, então:

    log 3(2) + log 5² (2)

    Aplica mais uma propriedade que diz que o expoente do logaritmando multiplica o log,então

    log 3(2) + 2. log 5 (2)

    Substitua pelo que o enunciado nos deu e temos:

    1/-1 . (a + 2b)

    como 1/-1 = -1, então multiplicamos tudo por -1

    assim obtendo:

    Gabarito letra E) -a - 2b

  • https://www.youtube.com/watch?v=wLHe50_IjOI


ID
980596
Banca
FUNCAB
Órgão
PM-ES
Ano
2013
Provas
Disciplina
Matemática
Assuntos

Sendo log 2 = 0,30 e log 3 = 0,47, o conjunto solução da expressão E = log 2.108 + log 3.10- 5 é:

Alternativas
Comentários
  • E= log 2+ log10^8 +log3+log10^-5

    E= log2 + 8 log10 + log3 +( -5 log10)

    como log10 =1, temos por substituição:

    E= 0,30+ 8(1) +0,47 + [(-5)(1)]

    E=0,30 +8 +0,47 -5

    E=3,77

  • log2+log10⁸+ log3+log10⁻⁵

    0,3+(8.1) + 0,47 + (-5.1)

    8,3+ ( -4,53 )

    8,3 - 4,53 = 3,77.

    LETRA E

    APMBB


ID
993646
Banca
Makiyama
Órgão
CPTM
Ano
2011
Provas
Disciplina
Matemática
Assuntos

Uma bactéria se espalhava no ambiente em que estava seguindo uma função logarítmica F(x) = log2 x (x >1), em que x é o tempo medido em minutos e F(x) é a área que possui a presença da bactéria em m². Após 32 minutos, a área ocupada será de:

Alternativas
Comentários
  • log(2) 32

    log(2) 2^5 = 5

    RESPOSTA LETRA E

     

  • e-

    log2(32) === 2^x=32.

    2^5=32


ID
1023235
Banca
PUC - SP
Órgão
PUC - SP
Ano
2011
Provas
Disciplina
Matemática
Assuntos

Considerando as aproximações log 2 = 0,30 e log 3 = 0,48, o menor número inteiro que satisfaz a sentença 10 n-1 > 135 15 está compreendido entre:

Alternativas
Comentários
  • Vou fazer bem detalhado para que todos entendam:

    10^(n - 1) > 135^15

    10^n . 10⁻¹ > 135^15

    Log 10^n - Log 10 > 15 Log 135

    n -1 > 15 Log 3³.5

    n -1 > 45 Log 3 + 15 Log (10/2)

    n -1 > 45.0,48 + 15 (Log 10 - Log2)

    n -1 > 21,6 + 15 .(1 - 0,2)

    n > 1 + 21,6 + 12

    n > 34,6

    Logo, o menor inteiro tá entre 25 e 35

    GABARITO: LETRA C


ID
1034662
Banca
UNIFAL-MG
Órgão
UNIFAL-MG
Ano
2008
Provas
Disciplina
Matemática
Assuntos

Analise as assertivas e assinale a alternativa correta.

I. √5 . 5√54 = 5 10√53.
II. Considerando log3 = 0,48 e log5 = 0,7, o valor de log0,75 é - 12
III.    7      _       5     = 1+ 2√3.
    3-√3         3+√3

Alternativas

ID
1065577
Banca
INEP
Órgão
ENEM
Ano
2013
Provas
Disciplina
Matemática
Assuntos

Em setembro de 1987, Goiânia foi palco do maior acidente radioativo ocorrido no Brasil, quando uma amostra de césio-137, removida de um aparelho de radioterapia abandonado, foi manipulada inadvertidamente por parte da população. A meia-vida de um material radioativo é o tempo necessário para que a massa desse material se reduza à metade. A meia-vida do césio-137 é 30 anos e a quantidade restante de massa de um material radioativo, após t anos, é calculada peta expressão M(t) = A . (2,7)kt , onde A é a massa inicial e k é uma constante negativa.

Considere 0,3 como aproximação para log102.

Qual o tempo necessário, em anos, para que uma quantidade de massa do césio-137 se reduza a 10% da quantidade inicial?

Alternativas
Comentários
  • tu iria gastar uns 10 minutos só nessa questão...


  • Questão simples, dá pra fazer por eliminação. 

    Suponha que você tem 100g de césio radioativo. Você quer que apenas 10g desse césio emitam radiação.
    Então deixa passar 30 anos, você terá 50g. Mais 30 anos, 25 g. Mais 30 anos, 12,5g. Até aí já foram 90 anos, mas você ainda não atingiu o objetivo. Mas já dá pra eliminar 4 alternativas, sobrando apenas 1 opção, a resposta.

    Resposta: Letra E


  • parabéns lucas. vc teve um pensamento muito produtivo nessa questão. eu gatei 15 mnts na mesma e quase não cheguei ao resultado.  graças a Deus obtive 100 como resposta. rrs abraço


  • 1Meia Vida=50% da massa inicial
    2MV= 25% da massa inicial
    3MV=12,5% da massa inicial

    A questão pede o tempo para obtermos 10% da massa inicial. Se para obtermos 12,5% da massa incial foram necessárias 3MV = 3x30anos = 90 anos, então para obtermos 10% da massa inicial seria necesário um pouco mais de tempo. A questão foi boazinha e só deixou 1 opção com mais de 90 anos. Caso houvesse mais, teríamos que usar um pouco mais de cálculos.


  • Essa é uma questão muito complexa e é sempre bom falar que pode ser resolvida por lógica uma vez que à cada meia vida temos metade da massa inicial então para chegar próximo 10% temos:

     

    100%/2 -----> 50%/2 -----> 25%/2 -----> 12,5%

    São pouco mais de 3 meias vidas,ou seja...quase 100 anos.

    Agora a resolução real:

    Se temos de pois de 30 anos metade da massa então:

     

    A/2=A(2,7)^30k

    1/2=2,7^30k

    30k=log(2,7)1/2

    Podemos fazer a mudança da base:

    30k=log1/2//log2,7

    Aplicando as propriedades temos:

    30k=log1 - log2//log3^3 - log10

    30k= -log2//3log3 - 1 

    30k= -0,3//3.0,48 - 1

    30k= -0,3//1,44 -1

    k= -0,3//0,44.30

    k= -0,3//13.2

    k= -0,02

     

    Agora que sabemos o valor da constante,podemos calcular o que o exercício nos pediu:

     

    A/10=A(2,7)^-0,02t

    1/10=2,7^-0,02t

    -0,02t=log(2,7)1/10

    -0,02t=log1/10//log3^3 - log10

    -0,02t= log1 - log10//0,44

    t= -1/0,44.(-0,02)

    t=(aprox) 103,3 anos

     

    R=Letra "E"

  • rapaiz nem perdi meu tempo com logaritimo fui logo na logica ta loko

  • 30 => 1/2

    60 => 1/4

    90 => 1/8

    10% => 1/10 mais de 90

    Letra E

  • Se uma questão dessa na prova, você não conseguir fazer por lógica já pula e deixa pra tentar no final...essas questões que cai em log ou exponencial(algumas) é certeza que mais da metade dos participantes não irão acertar, logo vai ser considerada difícil. Se tu perder tempo tentando fazer, vai pontuar pouco e o tempo que leva pra fazer uma questão dessa daria pra fazer 3 fáceis.

    É bom ficar ligado e tentar levar a questão pela lógica, bons estudos!!

  • Dá para montar uma equação através de acordo com a meia vida:

    Se em 30 anos a massa inicial cai pela metade:

    M(t) = Mo . 1/2^n (n é o número de períodos de 30 anos)

    Sabe-se que o examinador quer o tempo em que a massa inicial se reduzirá a 10% da massa inicial, logo:

    M(t) = Mo.10%

    substitua na equação:

    Mo.10/100 = mo.1/2^n

    10/100 = 1/2^n

    10/100 = 2^-n

    como não há posibilidade de igualar as bases, utilizaremos logaritimos:

    log(10/100) = log 2^-n

    1-2 = -n . log 2

    -1 = -0,3n x(-10)

    10 = 3n

    n = 10/3 ciclos

    Como podemos perceber, há 10/3 ciclos de 30 anos até a substância se reduziar a 10% da massa inicial, logo:

    10/3 x 30 = 300/3 = 100 anos

    Destarte, a substância leva 100 anos para reduzir até 10% da massa inicial.

  • Tempo: 0 Massa: 100%

    Tempo: 30 Massa: 50%

    Tempo: 60 Massa: 25%

    Se vai precisar de mais de 60 anos, só pode ser a alternativa E.


ID
1075390
Banca
CESGRANRIO
Órgão
Petrobras
Ano
2010
Provas
Disciplina
Matemática
Assuntos

Se b é um número real positivo, diferente de 1, logo, deduz-se que

Alternativas
Comentários
  • Para resolver, basta conhecer 2 propriedades do log: logaritmo do produto e logaritmo da potência.


    Logaritmo do produto:

    Se 0 < a ≠ 1, b > 0 e c > 0, então loga(b.c) = loga b + loga c.

     

    Logaritmo da potência:

    Se 0 < a ≠ 1, b > 0, então  loga(b^n) = n . logab


    Resolução:

    c) logb 24 = ( logb 3) + 3 (logb 2 )

    logb 24 =  logb 3.8 = logb 3 + logb 8 = logb 3 + logb 2³ = logb 3 + 3.logb 2

  • O que tem de errado na alternativa "A"?

  • Questão deveria ser anulada - 2 alternativas corretas!
    A letra A e C estão corretas dadas as condições do enunciado!!

     

     

  • Se b é positivo, diferente de 1, e b > 1 -> logb7 > log b 3
    Mas se b é positivo, diferente de 1, e 0 < b < 1 -> logb7 < logb 3

    Por isso letra a está errada, b pode assumir valores entre 0 e 1.


ID
1081225
Banca
Aeronáutica
Órgão
EPCAR
Ano
2013
Provas
Disciplina
Matemática
Assuntos

Pesquisas realizadas verificaram que, no planeta Terra, no início do ano de 2013, a população de pássaros da espécie A era 12 vezes a população de pássaros da espécie B.
Sabe-se que a população de pássaros da espécie A cresce a uma taxa de 5% ao ano, enquanto que a população de pássaros da espécie B cresce a uma taxa de 20% ao ano.
Com base nesses dados, é correto afirmar que, essas duas populações de pássaros serão iguais

(Considere: log 7 =0,85; log 6= 0,78; log2= 0,3)

Alternativas
Comentários
  • Cabulosa!!!!!!!!!!!!!!!!!!!!

  • https://www.tutorbrasil.com.br/forum/viewtopic.php?t=33001&__cf_chl_jschl_tk__=3cf6143d6d9fac7488d302d62acaa10e195741de-1589840020-0-AcsWQhY4RsrmvLyPcOJ09RfkTneGPjEHXQhR4QmKiDuZ9cwfRzH1RTUA3XdLtVrOAN4IUHB7N9lbAGIwzz7n_maTcTq03nTKTPPHHkOAh-Y1tMhr_RL63i_NCxQBkyUq8Ljuwy5ZxPWFa8U2oRVFlb7YRRU6_o-T33VPzPdR4pPRZs5KnecWt4jArAO5xEB7IanhZeVOa5HQEKpbkWYQu4GN9CEuu5q00yi8uY-MV_F1ljF3olJYPvaRdA5V1_XC-fBq9NDtaLKgWY8Q4qntP4fQhCsIZRA4PL98MXqF0PaVIYT1r_BgDW8hk7awIcoAlQ

    Esse é o link para quem quiser visualizar e melhorar !


ID
1100128
Banca
UERJ
Órgão
UERJ
Ano
2012
Provas
Disciplina
Matemática
Assuntos

Um lago usado para abastecer uma cidade foi contaminado após um acidente industrial, atingindo o nível de toxidez T0 ,correspondente a dez vezes o nível inicial.

Leia as informações a seguir.

• A vazão natural do lago permite que 50% de seu volume sejam renovados a cada dez dias.

• O nível de toxidez T(x) , após x dias do acidente, pode ser calculado por meio da seguinte equação:

T(x) = T0 . (0,5) 0,1x

Considere D o menor número de dias de suspensão do abastecimento de água, necessário para que a toxidez retorne ao nível inicial.

Sendo log 2 = 0,3, o valor de D é igual a:

Alternativas
Comentários
  • T(X)=to(0,5)^(0,1x)

    10Ti(nível de toxidez inicial)=to

    to/10=Ti

    to/10=to*(0,5)^(0,1x)

    1/10=0,5^(0,1x)

    10^(-1)=2^(-0,1x)

    10=2^(0,1x)

    log(2)=0,3

    log base(2)^(0,1x*0,3), logaritimando 2 = 0,3

    2^(0,03x)=2

    0,03x=1

    x=33,333-----------( 34 dias)

  • Questão difícil, pois tem muito dado para confundir:

    1) Dados desviante:" A vazão natural do lago permite que 50% de seu volume sejam renovados a cada dez dias."

    # Informação totalmente irrelevante

    2) Foco: "atingindo o nível de toxidez To ,correspondente a dez vezes o nível inicial."

    # Conclusão: Ti = 10 * To

    3) Cálculos:

    # A sacada aqui é fazer o lago voltar ao que era antes

    # Expressão: T(x) = To * 0,5 ^ (0,1x)

    # Lógica: Ti = 10 * Ti * 0,5 ^ (0,1x)

    4)

    # Ti = 10 * Ti * 0,5 ^ (0,1x)

    # 1 = 10 * 0,5 ^ (0,1x)

    # log(1) = log(10 * 0,5 ^ (0,1x))

    # 0 = log(10) + log(0,5 ^ (0,1x))

    # 0 = 1 + 0,1x * log(0,5)

    # -1 = 0,1x (log 5 * 10 ^ -1)

    # -1 = 0,1x [(log 5 + log(10 ^ -1)]

    # -1 = 0,1x [ log(10/2) + (-1)]

    # -1 = 0,1x [ log10 - log 2 - 1]

    5) Final

    Resposta: x = 33,333

    Resposta final = 34 dias para voltar a taxa inicial de toxidez


ID
1121641
Banca
ESAF
Órgão
MF
Ano
2014
Provas
Disciplina
Matemática
Assuntos

Sabendo-se que log x representa o logaritmo de x na base 10, calcule o valor da expressão log 20 + log 5.

Alternativas
Comentários
  • Dada a propriedade do produto:

    Log 20 + Log 5 = Log 20 . 5 = Log 100 = 2

  • LETRA D


    <meta property="og:image" content="http://qconcursos.com/assets/internas/compartilhar/geral-qc.png"/>

    Bons estudos a todos nós! SEMPRE!

    ..

  • Ao calcular o valor da expressão log 20 + log 5 temos:

    log 20 + log 5 = log(2.10) + log5 = log 2 + log 10 + log 5 = log 2 +  log 5 + log 10 =

    = log(2.5) + log 10 = log 10 + log 10 = 2.log 10

    Aplicando a definição temos log 10 = 1 e então     2.log10    =  2 (Resposta letra "D")


  • Fórmula:

    Log x a + Log x b = Log x a.b

  • Sabendo que:

    Log (m . n) = Log m + Log n, ou

    Log m + Log n = Log (m . n)

    Então:

    Log 20 + Log 5 = Log 20 . 5 = Log 100 = 2

    Obs:

    Log 10 = 1

    Log 100 = 2

    Log 1000 = 3 ...

  • Esta questão requer que o candidato demonstre conhecimentos sobre função logarítmica e suas principais propriedades.

    E = log20 + log5

    E = log(2 x 10) + log5

    E = log2 + log10 + log5

    E = log10 + log (2 x 5)

    E = log10 + log10

    E = 2 log10

    E = 2

    Resposta D)


  • Log da soma é igual ao produto dos logs

    Log (20.5) = Log 100 = 10^ x  = 10^2   --->  x =2

  • Fiz a resolução dessa questão em vídeo, veja:

    https://www.youtube.com/watch?v=qhtNYDOPSEs

  • Uma maneira inteligente de se ver a questão mesmo desconhecendo as propriedades dos logaritmos é pensar que

    log 10 + log 10 = 1 + 1 =  2, 10^2 = 100 portanto log 100 = log 10 + 10
  • É só utilizar a propriedade, onde log 20 + log 5 =?

    Ambos estão em bases décimais. 

    log 20x5

    log100=2

  • Na duvida galera, da pra fazer assim também:
    Log 20 é o mesmo que log 10*2, log de 10*2 é o mesmo que log 10 + log 2. Okay?
    Repete o processo no 5.
    Log 5 é o mesmo que log  de 10/2(pois 10/2 é 5), log 10/2 é o mesmo que log 10 - log 2.

    Joga na formula:
    Log 10 + Log 2 + Log 10 - Log 2. 

    Log 10 é 1, então :
    1 + Log 2 + 1 - Log 2.

    + log 2 e - log 2 se cancelam, sobra:
    1+1 = 2.

    Parece mais difícil mas pra mim saiu até mais fácil pois consegui fazer sem ver os comentários. 

  • https://www.youtube.com/watch?v=FX-gY07Kc-Q

  • Vamos “forçar” o aparecimento do número 10, pois sabemos que log10 = 1 (ou seja, o logaritmo de 10 na base de 10 é igual a 1, pois 10 = 10):

    log 20 + log 5 =

    log(2x10) + log(10/2) =

    (log2 + log10) + (log10 – log2) =

    log2 + 1 + 1 – log2 = 2

    Resposta: D

  • log com base vazia,a base e 10

    log20+log5

    log(20.5)

    log100

    R=2

  • Log 20 + Log 5

    Log 20 . 5= x

    10 ^ x = 20 . 5

    10 ^ x= 100

    10 ^ x= 10 ^ 2 (Se as bases são iguais, os expoentes são iguais).

    x = 2

    Gabarito Letra D

  • O logaritmo de um produto de dois números é igual à soma dos logaritmos

    log20+ log 5 = log(20*5)

    log100

    a base o enunciado diz que é 10

    10^x=100

    10 ^x=10²

    x=2

    LETRA D

    APMBB

  • Essa foi simples.

    Log de 20 x log 5= 100

    A base 10 elevada a quanto vai ser = 100

    Ou seja, 10²= 100

    2


ID
1151770
Banca
FUMARC
Órgão
CBM-MG
Ano
2013
Provas
Disciplina
Matemática
Assuntos

Para todos os números reais x e y, é CORRETO afirmar que

Alternativas
Comentários
  • Sabemos pelo enunciado que x e y são reais, e que a função f(x) =\log_ax é definida para a>0, a \neq 1 e x>a, assim, analisando cada alternativa:


    (a) \log x^2=2 \log x se e somente se x>0

    (b) \log(xy)=\log(x)+\log(y) se e somente se x>0 e y>0

    (c) \log(2+x^2)^2

    Observe que para todo x \in \mathbb{R}2+x^2>0. Então:

    \log(2+x^2)^2=2\log(2+x^2) \forall x \in \mathbb{R}

    (d) \log(x/y)=\log(x)-\log(y) se e somente se x>0,y>0



    Resposta: Alternativa C.
  • Qual é o erro das outras alternativas?

     

  • Qual é o erro das outras alternativas? Pra mim,todas estao corretas

  • Um prorpiedade simples, mas que quase ninguem sabe. 

    NÃO EXISTE LOG DE NÚMERO NEGATIVO.

    Como os numeros negativos estão contidos nos numeros reais, então temos que considera-los. 

    a - ERRADO - Não é possivel visto que não existe log de numero negativo

    b -  ERRADO - Não é possivel visto que não existe log de numero negativo

    c - CORRETO - O resultado de um numero negativo elevado a um exponte par é sempre positivo. Logo, essa é possivel.

    d -  ERRADO - Não é possivel visto que não existe log de numero negativo

  • As alternativas das questões se trata claramente das propriedades operativas dos logaritmos, nota-se que, no enunciado ela diz "PARA TODOS OS NÚMERO REAIS X E Y" e é aí que está a brincadeira, as propriedades dos logaritmos funcionam para todos os números reais positivos e não TODOS inclusive os negativos.

    Com esse raciocínio marquei a alternativa que não se trata de uma propriedade operativa dos log, portanto, a letra C


ID
1152541
Banca
UFGD
Órgão
UFGD
Ano
2010
Provas
Disciplina
Matemática
Assuntos

Uma empresa de derivados químicos considera que, quando x milhões de dólares são investidos em pesquisas, o lucro anual, em milhões de dólares, passa a ser

L ( x ) = 20 + 5Log 3 ( x + 3 )

De quanto deveria ser o investimento em pesquisa para que o lucro anual fosse de 40 milhões de dólares?

Alternativas
Comentários
  • L(x)= 20+5log3(x+3)

    40=20+5log3(x+3)

    40-20= 5log3(x+3)

    20=5log3(x+3)

    20/5= log3 ( x+3)

    4=log3 (x+3)

    3⁴=x+3

    x+3=81

    x=81-3

    x= 78

    78 MILHÕES

    LETRA C

    APMBB

  • fiz assim:

    5log 3/3 = 1 . 5 = 5

    20 + 5 ( x + 3 ) = 25 (x + 3)

    25 + 3 = 28

    28 + 40 = 78


ID
1155133
Banca
FUNCAB
Órgão
PM-RO
Ano
2009
Provas
Disciplina
Matemática
Assuntos

Apopulação de uma cidade cresce a uma taxa fixa de 5%ao ano. No ano de 1990 essa população era de 10.000 habitantes. A população desse país atingirá, aproximadamente, 43.200 habitantes no ano de:


(informação: log 432 2,635 e log 105 2,021)

Alternativas
Comentários
  • 432=100*1,05^t

    log432=log105

    2,635=2,021t

    t=30,23

     

    1990+30=2020. Letra B.

  • 4000/200=2000+20= 2020

    GAB B


ID
1196545
Banca
IBFC
Órgão
SEAP-DF
Ano
2013
Provas
Disciplina
Matemática
Assuntos

Supondo que o valor de uma máquina sofra uma desvalorização de 14% ao ano, a expressão que representa o tempo t em que o valor da máquina se reduzirá a um quarto do valor inicial é:

Alternativas
Comentários
  • Alguém saberia a resolução desta questão???? rsrsrs.... #confusão



    Até acertei a questão, mas fui pela lógica, e não por resolução matemática "concreta"... 



    rsrs

  • Para resolver esta questão temos que fazer uso da formula de depreciação de um produto.

    Vd=Vp (1-i)^t

    como Vd=1/4 Vp, temos 1/4vp=vp(1-i)^t  aplicando logaritmo teremos a resposta D.

  • Valor Contábil (VC) da máquina é uma função do tempo, pois sofre depreciação.

    No período em que t=0, ou seja, no início, VC = 100% = 1. Após, VC(t) = (100% - 14%)^t = 0,86^t.

    Queremos determinar o instante em que VC seja somente 25%, ou seja, tenha sofrido uma depreciação de 75% já. Então, VC(t) = 25% = 0,86^t = 0,25.

    Se aplicarmos log aos dois lados da igualdade, temos que log (0,86^t) = log 0,25. Como t é potência dentro do log, podemos "passá-lo pra fora". Fica assim: t . log 0,86 = log 0,25.

    Por fim, isola-se t.

    t = (log 0,25) / (log 0,86). Resposta D.  

  • o valor da maquina no primeiro ano será V0(1-0,14) (onde V0 é o valor inicial)

    no segundo ano ela sofrera nova desvalorização de 14% sobre o valor do ano anterior, ou seja V0(1-0,14)(1-0,14) = V0(1-0,14)^2

    assim podemos concluir que a função de desvalorização em função de um tempo em anos t  será F(N) = V0(1-0,14)^t

     

    como o problema quer saber o tempo em que o valor será 1/4V0 então temos:

    1/4V0 = V0(1-0,14)^t

    1/4 = 0,86^t

    0,25 = 0,86^t

    aplicando log dos dois lados temos:

    log 0,25 = log 0,86^t

    log 0,25 = t.log 0,86

    t = (log 0,25)/(log 0,86)


ID
1216306
Banca
CESGRANRIO
Órgão
Petrobras
Ano
2014
Provas
Disciplina
Matemática
Assuntos

Quanto maior for a profundidade de um lago, menor será a luminosidade em seu fundo, pois a luz que incide em sua superfície vai perdendo a intensidade em função da profundidade do mesmo. Considere que, em determinado lago, a intensidade y da luz a x cm de profundidade seja dada pela função y = i0 . ( 0,6 ) x/88, onde i0 representa a intensidade da luz na sua superfície. No ponto mais profundo desse lago, a intensidade da luz corresponde a i0/3

A profundidade desse lago, em cm, está entre.

Dados
log 2 = 0,30
log 3 = 0,48

Alternativas
Comentários
  • Quando nao fala nada da base ...que base e' ?     DEZ !!!!!

    onde  y = i (zero) * 0,6 exp (x/88) 

    entao :

      i(zero) / 3 = i (zero) * 0,6 exp (x/88)

      (i / 3) * (1/ i) = 0,6 exp (x/88)

      1/3 = 0,6 exp (x/88)

      log 1/3 = log 0,6 exp (x/88)

      === log 1 - log 3 = x/88 * log 6/10 

      ====  0  -  0,48 = x/88 * log 6/10

      88 * (- 0,48) = X * [ log 6 - log 10 ]         

                                          6 = 3 * 2   ===>   log 3 + log 2

                                           como log10 na base 10 = 1.

      - 42,24 = X * [ log 3 + log 2 - (1)]

      - 42,24 = X * [ 0,48 + 0,30  -  1 ]      ===>   X = - 42,24 / - 0,22   

      X = (42,24 / 0,22) =  192

                                                                      .:.         X = 192 cm 

  • Bem, se no ponto mais profundoi do lago, y=i0/3, então:

    i0/3 = i0 . (0,6) elevado a x/88

    1/i0 . i0/3 = 0,6 elevado a x/88

    1/3 = 0,6 elevado a x/88

    Se b elevado a x = a, dá, log a na base b = x, então:

    log 1/3 na base 0,6 = x/88 - 1º: log da divisão

    log 1 na base 0,6 - log 3 na base 0,6 = x/88 - 2º: log de 1 em qualquer base é = 0; 3º: mudança de base (para base 10)

    - log 3 / log 0,6 = x/88

    -log 3 / log 6/10 = x/88 - 4º: outro log da divisão

    - 0,48 / log 2.3 - log 10 = x/88 - 4º: log do produto; 5º: log 10 = 1

    -0,48 / (log 2 + log 3) - 1= x/88

    -0,48 / (0,30 + 0,48) - 1 = x/88

    -0,48*88 = (0,78 - 1) x

    -42,24 = - 0,22 x

    x = 42,24/ 0,22

    x = 192

     

     

     

     

  • Isso caiu em conhecimentos básicos?

  • Para quem achou confuso, vou explicar passo-a-passo:

    Lembrando que Y=i0/3 e Y=i0*(0,6)^x/88, então:

    y = i0 * (0,6)^x/88
    i0/3 = i0 * (0,6)^x/88, tá multiplicando, passa o i0 para o outro lado dividindo.
    i0/3/i0 = 0,6^x/88, duas divisões = multiplica a primeira pelo inverso da segunda
    i0/3 * 1/i0 = 0,6^x/88, corta os i0
    1/3 = 0,6^x/88, nesse momento, pode fazer duas coisas: ou jogar na propriedade de logaritmo mesmo ou logaritimizar (não sei o nome que se dá para isso) os dois lados. Acho que o segundo é mais fácil. Sempre na base 10.
    log 1/3 = log 0,6^x/88, propriedade de logaritmo com divisão. log 1/3 => log 1 - log 3
    log 1 - log 3 = log 0,6^x/88, todo log 1 = 0 e o log 3 = 0,48 como foi dito lá em cima.
     0 - 0,48 = log 0,6^x/88, propriedade de logaritmo com exponencial: se log tem exponencial, passa o bicho para o lado do log multiplicando, log 0,6^x/88 => x/88 * log 0,6
    0 - 0,48 = x/88 * log 0,6, esse 88 tá dividindo, passa para o outro lado multiplicando.
    -0,48 * 88 = x * log 0,6, aí é raciocínio lógico: log 0,6 => log 6/10, que é a divisão, joga na propriedade log 6 - log 10
    -42,24 = x * (log 6 - log 10), coloca entre parênteses para não se confundir. Todo log 10 = 1
    -42,24 = x * (log 6 - 1), outro raciocínio lógico, Log 6 => Log 3.2, que é a propriedade de multiplicação, ou seja, Log 3.2 = Log 3 + Log 2
    -42,24 = x * (log 3 + log 2 - 1), o enunciado disse os valores desses logs.
    -42,24 = x * (0,48 + 0,30 - 1), agora é continha básica.
    -42,24 = x * -0,22
    -42,42 / -0,22 = x
    x = 192

  • Putz, por isso sempre dava errado a conta, o enunciado eu interpretei sempre o (0.6) como multiplicação, tá de sacanagem,  y = i0 . ( 0.6 ) x/88 

  • Prova pra NASA?

  • lver esta questão o candidato deve:

    a) entender que a equação dada possui duas variáveis (x e y). O enunciado fornece o valor de y. Basta encontrar o valor de x;
    b) utilizar recursos algébricos para utilizar log 2 e log 3, ambos fornecidos no enunciado;
    c) relembrar as principais propriedades da Função Logarítimica.

    Assim,
    y = io . 0,6x/88
    Como y = io/3, tem-se:
    i0/3 = i0 . 0,6x/88
    1/3 = 0,6x/88
    log1/3 = log0,6x/88
    log1 - log3 = x/88 . log0,6
    0 - 0,48 = x/88 . log6/10
    (-0,48)(88) = x . (log6 - log10)
    -42,24 = x (log2 + log3 - 1)
    -42,24 = x (0,30+0,48-1)
    -42,24 = x(-0,22)
    x = 192

    Resposta E)

  • essa foi foda...

  • Y = i * (0,6)^x/88

    i/3 = i * 0,6^x/88

    Corta os " i "

    1/3 = 0,6^x/88

    "Transforma" em logaritmo

    log(1/3) = log(6/10)^x/88

    log 1 - log 3 = x/88 * (log 6 - log 10)

    log 1 - log 3 = x/88 * (log 2 + log 3 - log 10)

    0 - 0,48 = x/88 * (0,30 + 0,48 - 1)

    -0,48 * 88 = x * (-0,22)

    -42,24 = -0,22x

    Multiplique por (-1)

    42,24 = 0,22x

    x = 42,24/0,22

    x = 192

     

    A profundidade desse lago, em cm, está entre:     Alternativa (E) 190 e 200

     

  • Galera, todos resolveram muito bem porém tem um macetinho que facilitaria o final da conta que é a parte que fica:
    88*(-0,48) = X * 0,22

    Voces passem o 0,22 dividindo:
    88*(-0,48)/0,22 = X

    Multipliquem o 0,22 por 100 e o 0,48 também, vamos ter :
    88*(-48)/22 = X

    Agora dividam o 88 pelo 22:
    4*(-48) = X = -192, facilita bastante, nao precisa efetuar aqueles calculos com numeros quebrados.
     

  • 0,6^x/88 = 1/3 --> transformem logo esse  1/3 em 3 -¹,quando passar eles para a forma logaritimica ele vai ficar logo de 3 elevado -1. depois apliquem a mudança de base para os logs. 

       0,6^x/88 = 3 ^-1

             log 3^-1 /log 0,6 = x/88 ---------> usa a propriedade da potencia no log e arrasta o -1 la para a multiplicação. e o 0,6 transforma para 2x3/10,

    -1xlog 3/log 2 x log 3 - log 10 =x/88 ---------> lembra que log de 10 (decimal) é igual a 1

    -0,48 /0,48 + 0,30 - 1      =   x/88  

    -048/ -0,22     =     x/88 

     0,22.x =0,48.88 

    x = 48/100x88x100/22 

    x    =   192 

     

     

    letra e 

  • Quem fez essa simplificação: 6/10 => 3/5 dançou, pois o examinador dá log de 3 e de 2, e não dá log de 5. kkkkkkkk Questão fácil, porém, com muito cálculo, que atrapalha até os candidatos mais bem preparados.

  • Questão bacaninha. Deu pra treinar os conceitos de lpg =D.

  • Pegando com calma e com a ajuda do gabarito eu entendi a questão. Realmente, dá pra treinar as propriedades de log com ela.


ID
1264450
Banca
FUNDEP (Gestão de Concursos)
Órgão
COPASA
Ano
2014
Provas
Disciplina
Matemática
Assuntos

O conjunto solução da equação log(4. x + 2) = log ( 3x + 3 ) é:

Alternativas
Comentários
  • Log(4x+2)=Log(3x+3) 
    Log(4x+2)-Log(3x+3)=0 
    Usando uma propriedade que diz: 
    Log a/Log b = Log a - Log b,temos: 
    Log (4x+2)/Log (3x+3)=0 
    Vamos eliminar o logaritmo: 
    (4x+2)/(3x+3)=10º 
    (4x+2)/(3x+3)=1 
    4x+2=1(3x+3) 
    4x+2=3x+3 
    4x-3x=3-2 
    x=1 
    Solução: 
    Letra A

  • log (4. x + 2) = log ( 3x + 3 )

    sabemos q a base é 10 e o logaritmando é (4. x + 2)=( 3x + 3 )

    10^(4x+2)= 10^(3x+3)  

    bases iguais igualando os expoentes fica:

    4x+2 = 3x+3

    4x-3x = 3-2

    x = 1

    Gab A

  • O pessoal complicou demais, nossa!

    Se temos um log de mesma base dos dois lados da igualdade, o logaritmando também é igual, só isso:

    Equação de nível fundamental: 4x + 2 = 3x + 3, isso dá um x = 1

    Gab A


ID
1274050
Banca
CESGRANRIO
Órgão
LIQUIGÁS
Ano
2013
Provas
Disciplina
Matemática
Assuntos

A variável y, quando escrita em função de uma variável x, é dada por y = 10x+3 - 7.

A variável x, portanto, quando escrita em função da variável y, é dada por

Alternativas
Comentários
  • y = 10x³ - 7 => 10X³ = y + 7 => x³ = y+7/10; aplicando o log,  log x³ = log y+7 => 3log x = log y+7,

    log x = log y + 7 / log 3 => x = log (y +7) - 3, corrijam-me se estiver errado.

  • Marcelo está errado. Mas não foi culpa sua ! o 10 está elevado a X+3 , e não apenas 3. Até porque não faria sentido aplicar LOG se o X não está no expoente, basta tirar a raiz de terceira ordem.

  • y = 10 ^ (  x+3 ) - 7


    y = 10 ^ x . 10 ^ 3 - 7 


    ( y + 7 ) / 10^3 = 10 ^ x 


    log ( y + 7 ) / 10^3 = log  10 ^ x


    log ( y + 7 ) - log 10 ^ 3 = x  log 10


    log ( y + 7 ) - 3 =  x 

     



  • Trabalhando a função y = 10 x + 3 - 7, e colocando em função de "x", temos:


    y = 10 x + 3 - 7

    10 x + 3 = y + 7


    "Multiplicando" ambos os lados por log10:



    log10(10 x + 3) = log10(y + 7)


    Pela propriedade dos logaritmos, no primeiro membro da equação, como a base do log é igual a base do logaritmando, podemos eliminar o log, assim:



    log10(10 x + 3) = log10(y + 7)

    x + 3 =  log10(y + 7)

    x = log10(y + 7) - 3


    Resposta: Alternativa A.
  • Resposta: 


    y= 10 ^ x+3 -7 

    y + 7 = 10 ^ x+ 3 

    log ( y+7) = log 10 ^ x + 3   ...... os logs tem base 10! 

    log (y + 7) = x + 3  


    x= log ( y + 7) -3 

  • atenção! o enunciado diz 10x^+3 , porém o certo é 10^x+3

  • y = 10^x+3 - 7

    y+7 = 10^x+3

    log(y + 7) = x + 3

    x = log(y + 7) - 3

     Alternativa (A)

  • Vamos lá!

    y = 10^(x+3) -7

    y+7=10^(x+3)

    vamos aplicar algumas regrinhas aqui.

    note que a^(c+b)= a^b*a^c, então temos:

    y+7=10^x*10^3

    y+7= 10^x*1000..........

    (y + 7)/1000=10^x

    .vamos aplicar log de base 10 em ambos os membros:

    log{(y + 7)/1000}= log10^x.....

    bom, o log do quociente é igual a diferença dos logs, enão vamos aplicar essa propriedade no primeiro membro.

    log(y+7) - log1000 = log10^x

    log(y+7) -3 = log10^x.... basta fazermos o tombamento de x.

    log(y+7) -3= x.log10

    logo, x= log(y+7)

    espero ter ajudado!

  • y=10^(x+3)-7

    y+7=10(x+3)

    x = log(y+7)-3

  • Porque nao poderia ser correta a letra B, por exemplo?

     

    Se o Logy + Log7 pode ser reescrito como Log(y.7)

     

    Afinal:

     

    y=10^(x+3)-7

    Logy = x+3 - Log7

    Logy + Log7 - 3 = X    logo: X = Log(7.y) - 3 

     

    Porque nao? 

  • Major Tom, quando você coloca a expressão inteira em log, precisa ser o lado inteiro da equação, não elementos separados.
    Por exemplo:
    A + B = C só pode ser transformado em Log (A+B) = Log (C), e não Log A + Log B = Log C.



    Mas eu posso estar errado

  • Major Tom, se você fizer isso, vai estar dizendo que Y = Log Y, isso não é verdade.

  • y = 10^(x+3) - 7

    o 7 passa somando

    y+7 = 10^(x+3)

    Aplica log10

    log10(y + 7) = x + 3

    mostra quem X é

    x = log10(y + 7) - 3

    Alternativa "A"

  • caramba.. consigo chegar na A e na B

    não consigo enxergar pq a B estaria errada.

  • Erro da Alternativa B

     

    A propriedade é Log(a.b) = log(a) + Log(b) ou Log(a/b) = Log(a) - Log(b). Não existe a propriedade para Log(a+b). Então, no Item B, temos:

    b) x = Log (7y) - 3  = Log(7) + Log(y) - 3 [Errado, pois esse valor é diferente de x = Log(7+y) - 3]

  • y = 10^x+3 - 7.

    Log (y+7) - 3 = x

    O logaritmando é (y+7), ou seja, alternativa A, pois se não fosse seria então y+4.

    Mesmo assim, aquele parenteses pode levar ao engano de Log y + Log 7, o que daria à Log (7y), ou seja, Log (7y) -3 = x, ou seja, alternativa B.


ID
1354459
Banca
FUNCAB
Órgão
POLITEC-MT
Ano
2013
Provas
Disciplina
Matemática
Assuntos

Sabendo que o pH de uma substância líquida pode ser calculado, matematicamente, através da função pH = -log[H+], determine o pH de uma substância líquida cujo H+ = 0,000001.

Alternativas
Comentários
  • Fórmula =>  pH = -log[H+]

    Substituindo na fórmula fica:

    pH = -[ log 0,000001 ]

    pH = -[ log (1/1.000.000) ]

    pH = -[ log 1 log 1.000.000 =6 é só contar a quantidade de zeros que se obtem o valor

    Sabendo que log de 1 é zero fica:

    pH = 0 -[ -6 ]

    pH = 6

    Gab A

     


ID
1423723
Banca
FUNIVERSA
Órgão
SEGPLAN-GO
Ano
2015
Provas
Disciplina
Matemática
Assuntos

o analisar as causas da morte de um indivíduo, um perito laboratorial identificou, no fígado do cadáver, a presença de uma bactéria que, a cada minuto, dobrava em quantidade. O perito conseguiu separar uma dessas bactérias, colocou-a em um meio adequado e ela começou a se reproduzir. Considerando como t = 0 o instante em que a bactéria foi colocada no meio e que 0,3 seja o valor aproximado, para log 2, para que se atinja a quantidade de 1.000.000 de bactérias, é necessário que transcorram

Alternativas
Comentários
  • Letra B

    No momento inicial temos 1 bactéria. Após t = 1 minuto esse número émultiplicado por 2, ficando 2x1 = 21 = 2 bactérias. Após mais um minuto (ou seja,em t = 2 minutos) esse número é multiplicado por 2 novamente, ficando 2x2 = 22 = 4bactérias. Após mais um minuto (ou seja, em t = 3 minutos) esse número é multiplicado por 2 novamente, ficando 2x2x2 = 23 = 8 bactérias, e assim por diante.Assim, veja que o número de bactérias em um determinado minuto "t" igual a 2t. Para que o número de bactérias seja igual a 1.000.000, temos:  

    2t = 1.000.000 

    2t = 106log2t = log106

     t x log2 = 6 x log10

     t x 0,3 = 6 x 1

     t = 6 / 0,3 

    t = 20 minutos 

  • 2^t = 1000000 => 2^t = 10^6

    Como log2 = 0,3, tem-se que 10^0,3 = 2

    Voltando acima, substituimos o 2 pelo outro valor similar: 10^0,3xt = 10^6 

    Como estao na mesma base, resolvemos os expoentes e encontraremos o tempo:

    0,3 x t = 6 => t = 6/0,3 => t = 20

  • Eu faço assim (se ajudar a simplificar)

    A = 10 ^ 6 (quantidade final de bactérias)

    Ao = 1 (quantidade inicial de bactérias)

    2 ^ t = (a cada minuto, a bactéria duplica a quantidade a partir da inicial, assim, em um minuto, tem-se 2 bactérias; em 2 minutos, 4 bactérias.


    Deste modo:

    A = Ao x (2 ^t)

    10 ^6 = 1 x (2 ^ t)

    log (10 ^ 6) = log (2^ t)

    6 x log (10) = t x log (2)

    6 = t x 0,3

    t = 20 min

  • Quem foi dobrando os valores no papel clica aqui rsrsrs.

  • Pode imaginar como sendo uma progressão geométrica de razão 2  onde o primeiro termo é 1, o segundo é 2 e An = 1000.000 

    An = A1*q^n-2

  • O problema diz que a bactéria dobra a quantidade a cada minuto, logo:

    t=0 , 1 bactéria

    t =1, 2 bactérias

    t=2, 4 bactérias

    ...

    e assim sucessivamente ...

    sabemos que isso está em P.G


    an = a1 * q^n

    1.000.000 = a1 * 2^n


    Usando as propriedades do logaritmo vem:

    Log 1.000.000 na base 2 = n


    Passando para base 10:


    Log 1.000.000 / Log 2 = n

    Log2 = 0,3 (informado na questão)

    Log 1.000.000 = 6


    6/0,3 = n


    n = 20


    GABARITO: LETRA B)


    Bons estudos galera ..


ID
1445338
Banca
CONSULTEC
Órgão
PM-BA
Ano
2011
Provas
Disciplina
Matemática
Assuntos

Com a contínua evolução tecnológica, a cada dia os aparelhos eletrônicos são produzidos com processadores mais velozes, que conseguem realizar suas tarefas num tempo cada vez menor.

Supondo-se que o tempo, em milésimos de segundo (milissegundos), que certo componente eletrônico leva para processar xbits, seja dado por T(x) = log8 x e considerando-se log 2 = 0,30, pode-se concluir que 250bits serão processados em, aproximadamente,

Alternativas
Comentários
  • t(x) = log 8 ( x)                                                  10^x = 5

    t = log 8 (250)                                                    (10^0,3)^2 = 2^2  -> 10^0,6 = 4       pela regra de 3    x = aproximadamente 0,75 

    t= log 250 / log 8                                                

    t=  (log 25 * 10) / (log 2 ^3)

    t = (log 5^2 + log 10) / ( 3 log 2)

    t = (2 log 5 + 1) / 0,9                    

    t = ( 2 * 0,75 + 1 ) / 0,9

    t = 2,5 /0,9   t = aproximadamente 2,7       logo gabarito letra  a

  • log(5) = log10 - log2 = 0,7

    logo t = 2,4/0,9 = 2,66 aprox

  • Lembremos das regrinhas do logs

    log2= 0,3

    T(x)=log8x

    x= 250

    Log8(250), crie uma base comum aos dois , no caso a base 10.

    Log250/log8

    log(10.25)/log2³

    log25+1/3log2

    log5²+1/3.0,3

    2log5+1/0,9 »»» qual o log5?????

    [log5]

    log10/log2= log5, então 1-0,3= 0,7

    continuando...

    2.0,7+1/0,9

    2,4/0,9= 2,66 aproximadamente.

    LETRA A


ID
1447684
Banca
CESGRANRIO
Órgão
Petrobras
Ano
2015
Provas
Disciplina
Matemática
Assuntos

Sejam M = log 30 e N = log 300.

Na igualdade x + N = M, qual é o valor de x?

Alternativas
Comentários
  • M = log30, M = log10+log3

    N = log 300, N = log 10+log10+log3

    x = log10+log3-log10-log10-log3, x = -log10 ou -1

  • x+N=M


    x+log3+log100=log3+log10
    x+log3-log3+log100-log10=0
    x+log100-log10=0
    x+log(100/10)=0
    x+log10=0
    x+1=0
    X=-1...................letra B
  • Se M = log 30 e N = log 300

    Então, x+N=M ficaria:
    x+log300=log30

    Log300 é a mesma coisa que Log10+log30, então
    x+log10+log30=log30

    Log10 é igual a 1, então
    x+1+log30=log30
    x+1=log30-log30
    x+1=0
    x = -1

    Letra: B

  • Gabarito: B.

    Relembrando uma das propriedades dos logaritmos:

    log a - log b = log (a/b)

    Quando a base do logaritmo não é discriminada, considera-se como sendo de base 10, que é o caso dessa questão.

    Na questão:


    x = M - N; substituindo pelos valores de M e N:


    x = log 30 - log 300

    x = log (30/300)

    x = log (1/10) ---> 1/10 = 0,1 = 10^(-1)

    log 1/10 = log 10^(-1)

    log 10^(-1) ---> 10^? = 10^(-1) ---> ? = -1.


    Bons estudos!

  • Resolução da questão em Vídeo

    https://www.youtube.com/watch?v=9VY5WJjfgps
  • Resolvendo aplicando as propriedades do log:

    x + N = M

    x + log 300 = log 30

    x = log 30 - log 300

    x = log(30/300)

    x = log(1/10)

    x = log 1 - log 10    (Lembrando que log10 = 1)

    x = 0 - 1 

    x = -1


    Resposta: Alternativa B.
  • x+log300=log30

    x=log30-log300

    x=log30/log300

    x=log1/10

    x=log10^-1

    x=-1

  • X + log300 = log30                                        

    X = log30 – (log300)

    X = log30 –( log30.10)    Obs: na propriedade da multiplicação de log temos que uma multiplicação vira adição log30.10 = log30+log10

    X = log30 – (log30 + log10)

    X = log30 – log30 – log10

    X = - log10

    X = -1                                  R- Letra b

    Obs: log10 na base 10 = 1  ( log10 = x   ----------- 10ˣ = 10¹ --------X = 1)

     

     

  • Sejam M = log 30 e N = log 300.

    Na igualdade x + N = M, qual é o valor de x? 

    Então, Substituindo teremos:

    x + log 300 = log 30
    isolando o x, passa log 300 para o outro lado
    x= log 30 - log 300
    x= log 30 - (log 30 x 10) >> Na regra de log (produto) vira (soma).
    então:
                 x= log 30 - (log 30 + log 10)
                 x= log 30 - log 40
         x = - log 10
     

    R- Letra b

    Obs: quando ñ se tem a base admite-se que a base é 10.
      log10 na base 10 = 1  ( log10 = x   ----------- 10ˣ = 10¹ --------X = 1)

  • Eu fiz de outro jeito, acertei, porém não sei se está correto. Se algúem puder me dizer..

    x+n=m

    X+log300=log30 (aqui eu cortei um zero de cada lado, então ficou:)

    x+log30=log3 (dai fatorei o 30 iniciando a divisão por 10, entao ficou:)

    x+log10ˆ1 x 3ˆ1 = log3 (agora usei a propridade da multiplicacao de logs que vira soma)

    x+log10 + log3 = log 3 (agora vou isolar o x)

    x= -log10 - log3 +log3 (-log3 com + log3 fica zero, entao ficou:)

    x=-log10 (usando da propriedade da base 10 que é 1, ficou:)

    x= -1

  • Log de 3 é 0,48

    Log de 30 = Log de 3 (0,48) + Log de 10 (1)

    Log de 30 = Log de 3 (0,48) + Log de 100 (2)


    x = M - N

    x = 1,48 - 2,48

    x = -1


    #BrasilAcimaDeTudo

  • x+log300=log30

    x+log(3.100)=log(3.10)

    x+log3+log100=log3+log10

    x+0,48+2=0,48+1

    x+2,48=1,48

    x=1,48-2,48

    x=-1

    OBS:por ter resolvido questoes anteriores com log3,por experiência deduzi que log3=0,48.

    Não importa a cor do gato desde que ele pegue o rato.

    johan carl Friedrich gauss

  • Solução adequada.

    x = Log(3.10) - Log(3.10^2)

    x = Log[(3.10)/(3.10^2)] propriedade dos logs.

    x = Log(10/10^2)

    x= Log(10^-1) divisão de potência de mesma base, repete-se a base e subtraem-se os expoentes.

    10^x = 10^-1 definição dos Logs.

    x = -1

  • GAB [B] AOS NÃO ASSINANTES .

    #ESTABILIDADESIM.

    #NÃOÀREFORMAADMINISTRATIVA.

    ''AQUELES QUE , PODENDO FAZER SE OMITEM , SERÃO CÚMPLICES DA BARBÁRIE.''

  • Veja que:

    M = log30 = log(3x10) = log3 + log10 = log3 + 1

    N = log300 = log(3x10^2) = log3 + 2xlog10 = log3 + 2

    Assim, sendo:

    x + N = M

    Podemos substituir os valores conhecidos, ficando com:

    x + log3 + 2 = log3 + 1

    x + 2 = 1

    x = 1 - 2

    x = -1

    Resposta: B

  • Questão resolvida no vídeo abaixo.

    https://www.youtube.com/watch?v=Qhl6mWnAqH8

    Bons estudos.

  • EXCELENTE CANAL RESOLUÇÃO BEM EXPLICADA

    https://www.youtube.com/watch?v=b4KmhR0RvI8&t=1068s

    RUMO A APROVAÇÃO CONCURSO BANCO DO BRASIL!!!


ID
1454173
Banca
CESGRANRIO
Órgão
Petrobras
Ano
2015
Provas
Disciplina
Matemática
Assuntos

A estabilidade de um determinado processo industrial é avaliada a partir de um índice N, que é um número real positivo. O processo é considerado estável se, e somente se, 3 ≤  log3 (N) ≤ 4. O processo é dito instável se, e somente se, o mesmo não for estável.
Dessa forma, o referido processo industrial é considerado instável se, e somente se, o índice N pertence ao conjunto 

Alternativas
Comentários
  • resolução no link :

    https://s3-us-west-2.amazonaws.com/estrategia-blog/2015/03/Resolu%C3%A7%C3%A3o-das-provas-de-nivel-superior.pdf
  • Uma crítica ao link dado como resposta.
    O logb (y) = x, quer dizer: Qual o valor (x) que "b" elevado a ele resulta em y, pois y=b^x.

    Logo,
    Nesta questão: Y=N; b=3 e X=4, sendo assim:
    log3 (N) = 4, nos questiona qual é o valor de N que satisfaça 3^4.
    Seria 81, e não o valor 64 posto como resposta neste link citado.
    Tanto que a resposta certa é a letra b.

  • Log N na base 3 = 3; Temos N = 27

    Log N na base 3 = 4; ; Temos N = 81

    No comando da questão afirma que N é um múmero real positivo. Então temos apenas números maiores que zero.

    Estável é o processo que se encontra entre o intervalo N=27 e N=81

    Instável fora desses valores, o que implica em colchetes abertos.

    Portanto, temos ] 0, 27 [ ∪ ] 81, + ∞ [

    Letra B

  • No meu navegador só aparece o sinal de '=":   3 = log3 (N) = 4

    Quando na verdade é >=

    Tive que olhar a prova em pdf, pois não estava entendendo a questão por causa disso.

  • https://www.qconcursos.com/arquivos/prova/arquivo_prova/41522/cesgranrio-2015-petrobras-profissional-junior-administracao-prova.pdf

    esse é o pdf da prova aqui do site, questão 56, para conferir a formatação ;)


ID
1482133
Banca
Aeronáutica
Órgão
EEAR
Ano
2014
Provas
Disciplina
Matemática
Assuntos

Se a > 0, b > 0, c > 0 e c ≠1, então é correto afirmar que

Alternativas
Comentários
  • Pq é a letra C e não a B ?

  • A relação da B não existe Yngvar

  • Propriedade

    Log(a.b)=Log a + Log b

    letra C

  • não será a B pois está de + para x. e não pode, o que pode é de x se transformar em + (soma de dois logaritmos). resposta certa: letra C
  • LOGa B.C------- LOGa B + LOGa C TEM QUE ESTUDAR PROPRIEDADES PARA SABER

  • kkkk se não souber as propriedades de log, podem achar que é produtos notáveis como o colega abaixo

  • GABARITO - C

    É uma propriedade dentro dos logaritmos.

  • Confundi com a B


ID
1482412
Banca
Aeronáutica
Órgão
EEAR
Ano
2014
Provas
Disciplina
Matemática
Assuntos

Seja x um número real positivo e diferente de 1. Assim, logx 1 + log xx é igual a

Alternativas
Comentários
  • gabarito letra c

    logx1 = logaritmando igual 1 resulta em 0

    logxX= logaritmando igual a base resulta em 1

    logo, logx1 + logxX = 0+1= 1.

  • questão que devia ter sido anulada

  • VICTOR HENRIQUE, ESSA QUESTÃO E A MAIS FÁCIL DA PROVA

    LOGx1---- QUALQUER LOG QUE O 1 SEJA EXPOENTE E = 0

    LOGxX----- E A MSM COISA QUE LOG2 2=1

    0+1=1

  • Diferente de 1 que dá 1? Parece até o discurso da Dilma

  • Seja x um número real positivo e diferente de 1...

    uai, então posso fazer log2 1 + log 2 2 >> 0 + 1 =

  • O pessoal está lendo a questão erradamente..

    A questão diz que x≠1

    O resultado final que é 1 é o resultado da soma dos logaritmos, e não o valor de x

    A questão está certa, só precisa ter cuidado para ler com calma e atenção!


ID
1482841
Banca
CEPERJ
Órgão
SEDUC-RJ
Ano
2015
Provas
Disciplina
Matemática
Assuntos

Considere uma matriz A 3X3 , formada por elementos a ij que representam os logaritmos decimais de (i+j), isto é, a ij = log(i+j). Se log2 = 0,301 e log3 = 0,477, a soma dos elementos da diagonal principal da matriz A equivale a:

Alternativas
Comentários
  • LETRA B

    A soma da diagonal principal ficas assim:  LOG2 + LOG4 + LOG6

    LOG2 + LOG(2.2) + LOG(3.2)

    LOG2 + (LOG2 + LOG2) + (LOG3 + LOG2)

    0.301 + 0.301 + 0.301 + 0.301 + 0.477 = 1,681

     

  • Aij é um elemento da matriz onde i e j significam o posicionamento da linha e da coluna respectivamente.

    A soma da diagonal principal é dada por: a11 + a22 + a33

     

    a11 = log ( 1 +1 ) = log 2 = 0, 301

     

    a22 = log ( 2 + 2 ) = log 4 = log 22 = 2 x log 2 = 2 x 0,301 = 0, 602

     

    a33 = log ( 3 +3 ) = log 6 = log ( 2 x 3) = log 2 + log 3 = 0,301 + 0, 477 = 0,778 

     

    logo, a11 + a22 + a33 = 0, 301 + 0, 602 + 0,778 = 1,681

     

    resposta letra B

     Uns encurvam-se e caem, mas nós nos levantamos e estamos de pé.

    Sm: 20; 8

    YOU TUBE: PROF ROGERIO SILVA

    https://www.youtube.com/channel/UCjqMyxJqW98dkyOgIXBc1Ig?view_as=subscriber

    ROGERIO CONCURSEIRO: MAPAS MENTAIS E QUESTÕES

    https://www.youtube.com/channel/UC9jMABWHjXyzLdLGa-ziRTw?view_as=subscriber

  • A questão praticamente foi matriz. Sabendo assunto de matriz e a diagonal principal, já matava a questão.


ID
1529371
Banca
FUNCAB
Órgão
POLITEC-MT
Ano
2013
Provas
Disciplina
Matemática
Assuntos

Sabendo que o pH de uma substância líquida pode ser calculado, matematicamente, através da função pH = -log[H+], determine o pH de uma substância líquida cujoH+ = 0,000001.

Alternativas
Comentários
  • pH = - log[H+]

    pH = - log10⁻⁶

    pH = - (10⁻⁶ = 10^x)

    pH = - (- 6)

    pH = 6

    GABARITO: LETRA A

    Ou ,simplesmente, conte o número de casas depois da vírgula.


ID
1555366
Banca
UERJ
Órgão
UERJ
Ano
2015
Provas
Disciplina
Matemática
Assuntos

Admita que a ordem de grandeza de uma medida x é uma potência de base 10, com expoente n inteiro, para 10n-1/2x < 10n+1/2 .

Considere que um terremoto tenha liberado uma energia E, em joules, cujo valor numérico é tal que log10 E = 15,3.

A ordem de grandeza de E, em joules, equivale a:

Alternativas
Comentários
  • De acordo com a propriedade fundamental do logaritmo, podemos escrever log10 E = 15,3 como:
    E = 1015,3 
    Logo:
    1015 -1/2 ≤ 1015,3 < 10 15+1/2 
    10 15 - 0.5 ≤ 1015,3 < 10 15 + 0.5 
     n = 15 
    Então 1015,3 ≡ 1015

    Resposta: Alternativa B.
  • log10 E = 15,3 
    E = 1015,3
    Desse modo, tem-se:
    10n – 0,5  1015,3 < 10n + 0,5
    1015 – 0,5  1015,3 < 1015 + 0,5
    Conclui-se, então, que n = 15.
    A ordem de grandeza equivale a 1015.


    Gabarito B


ID
1583725
Banca
Exército
Órgão
IME
Ano
2011
Provas
Disciplina
Matemática
Assuntos

Se log102 = x e log10 3 = y, então log518 vale:

Alternativas
Comentários
  • Apesar das questões do IME serem bastante difíceis na sua maior parte, essa questão é considerada nível mediano. Quem já sabe trabalhar um pouquinho com Log conseguirá desenvolver essa questão!!! 

     

     

    BRASIL!!!!!

  • Log5 18= Log 18/Log 5


    Log 18/Log 5

    Log (3 . 3 . 2)/ Log (10/2) 

    2Log3+Log2/Log10-Log2

    2y+x/1-x


    Reposta: 2y+x/1-x

    letra (A)

  • BIZU É TROCAR A BASE E APLICAR AS PROPRIEDADES DE LOG

    LOG(A/B)=LOG A- LOG B

    LOG(A.B)=LOG A+ LOG B

    COM ESSAS PROPRIEDADES, CONSEGUE RESOLVER.


ID
1607779
Banca
OBJETIVA
Órgão
CISVALE
Ano
2015
Provas
Disciplina
Matemática
Assuntos

Qual deve ser o tempo, em anos, para que o capital de R$ 100.000,00 gere R$ 46.410,00 de juros, quando aplicado à taxa de 10% ao ano, no sistema de juros compostos?

(Usar: log 1,1 = 0,04 e log 1,4641 = 0,16)

Alternativas
Comentários
  • Capital = 100.000,00
    Juros = 46.410,00     +
    Montante = 146.410,00
    i = 10% ao ano
    log 1,1 = 0,04
    log 1,4641 = 0,16

    M = C . ( 1 + i ) ^ t   Fórmula Juros Compostos

    146.410 = 100.000 x (1 + i ) ^ t
     

    146.410 = ( 1 + 0,1 ) ^ t
    100.000

     

    1,4641 = 1,1^t

    log 1,4641 = log 1,1 ^t   Aplica o log que está no enunciado do exercício.

    log 1,4641 = t x log 1,1   Aqui a potência vai para frente do logarítmo, multiplicando.

    0,16 = t x 0,04    Não precisa calcular pois o enunciado já dá o valor dos log. Como tem um sinal de igual você pode simplificar os lados, por 4.

    T = 4

     

    FOCO, FORÇA, FÉ, FRANGO FRITO, FARINHA, FILÉ

  • Dá p resolver a questão usando a lógica também, pois levando em consideração que a taxa anual é 10%, em menos de 4 anos não conseguiríamos mais de 40k de rendimento e em 5 anos ou mais teríamos um rendimento superior a 50k, logo a única alternativa plausível é a B.

  • Era só lembrar que 1,1 na potência 4(anos) = 1,4641. 

  • No 1° ano foi para 110.000,00 (100+10)

    no 2° ano foi para 121.000,00 (110+11)

    no 3° ano foi para 133.100,00 (121+12,1)

    no 4° ano foi para 146.410,00 (133.1+ 13,1)

  • formula dos juros compostos:m=c(1+i)^t

    146.410 o montante

    100.000 o capital

    0,1% a taxa

    t o tempo

    resolve essa parte e chega na parte em que se aplica os logaritmos....

    1,1^t=1,4641

    log1,1^t=log1,4641

    t.log1,1=log1,4641

    t.0,04=0,16

    t=0,16/0,04

    t=4 anos

  • M = J + C

    M = 46410 + 100000

    M = 146410

    M = C*(1+ i)^t

    146410 = 100000*(1+ 10%)^t

    146410 = 100000*(1,1)^t

    146410/100000=1,1^t

    1,4641=1,1^t

    t= log 1,4641/log1,1

    t= 0,16/0,04

    t=4


ID
1614451
Banca
PUC - RJ
Órgão
PUC - RJ
Ano
2014
Provas
Disciplina
Matemática
Assuntos

Seja x = log23 + log29 + log227.

Então, é correto afirmar que:

Alternativas

ID
1614505
Banca
PUC - RJ
Órgão
PUC - RJ
Ano
2014
Provas
Disciplina
Matemática
Assuntos

Se log1/2x = -3, então 3√x  +x2 vale:

Alternativas

ID
1626559
Banca
UECE-CEV
Órgão
UECE
Ano
2013
Provas
Disciplina
Matemática
Assuntos

Se os números reais a e b são positivos, distintos, diferentes de 1 e satisfazem a igualdade bx = ax/h  para qualquer número real x, então, para n positivo e diferente de 1, o valor de h é

Alternativas
Comentários
  • bx = a^(x/h)

    log b * x = log a^(x/h)

    log b * x = (x/h) * log a

    log b * x = (x * log a)/h

    log b = (x * log a)/(h * x)

    log b = log a/h

    (Agora, é inverter log b com h) =>

    h = log a / log b


ID
1650196
Banca
UEPA
Órgão
PM-PA
Ano
2012
Provas
Disciplina
Matemática
Assuntos

Considere os inteiros positivos α, β , Υ, δ. Sabe-se que (β- 1). log α = log Υ e que α1-β = δ.

Nestas condições, em relação aos valores de Υ e δ é correto afirmar que:

Alternativas

ID
1713358
Banca
CONSULTEC
Órgão
PM-BA
Ano
2014
Provas
Disciplina
Matemática
Assuntos

Quando o número de queixas de roubo de aparelhos celulares registradas em uma delegacia chegou a 100, passou-se a monitorar essas queixas, constatando-se que o seu crescimento era, em média, de 20% a cada semana.

Nessas condições, considerando, se necessário, log2 = 0,31 e log3 = 0,48, pode-se estimar que o número de queixas semanais deverá ultrapassar 1200 em um número de semanas, no mínimo, igual a

Alternativas
Comentários
  • 1200=100.1,2^s

    12=1,2^s => log12 na base 1,2 = s => log 12 / log 1,2 = log (4.3) / log (4.3/10) = ( 2.log 2 + log 3 )/( (2.log 2 + log 3) - 1) =1,1/0,1=11 semanas 

    Ficar ligado na pegadinha.. para ultrapassar 1200 tem q ser maior do que 11 semanas, logo, 13 semanas é a opção mais próxima. 

  • Que sacanagem em ... tava já preocupado no que errei ... pegadinha das boas! valeu Bruno (y)

  • Considerar esse valor aí, log 2 = 0,31, te faz encontrar resposta equivocada. Atenção: log 2 = 0,30 (ou 0,301, com três casas decimais).


ID
1738699
Banca
FUNCAB
Órgão
CBM-RO
Ano
2014
Provas
Disciplina
Matemática
Assuntos

O conjunto solução da equação log4 (log9 3x3) = -1/2, é:

Alternativas

ID
1744690
Banca
Exército
Órgão
IME
Ano
2015
Provas
Disciplina
Matemática
Assuntos

Sabendo-se que os números reais positivos a, b e c formam uma progressão geométrica e log(5c/a), log(3b/5c) e log(a/3b) formam uma progressão aritmética, ambas nessa ordem, então pode-se afirmar que a, b e c

Alternativas

ID
1764868
Banca
FGV
Órgão
FGV
Ano
2015
Provas
Disciplina
Matemática
Assuntos

Sendo p e q números reais, com p>q e p+q>0, definiremos a operação # entre p e q da seguinte forma: p#q=p2 –q2 +log(p+q), com log(p+q) sendo o logaritmo na base 10 de (p+q). Utilizando-se essa definição, o valor de 10#(–5) é igual a

Alternativas
Comentários
  • p#q = p^2 - q^2 = log(p+q)

    Fazendo p = 10 e q = -5 tem-se:

    10#(-5) = 10^2 - (-5)^2 + log[10 + (-5)]

    10#(-5) = 100 - 25 + log 5

    10#(-5) = 75 + log 5

    10#(-5) = 75 + log(10/2)

    10#(-5) =  75 + (log 10 - log 2)

    10#(-5) = 75 + 1 - log 2

    10#(-5) = 76 - log 2  ===> GABARITO: C


ID
1766575
Banca
BIO-RIO
Órgão
ETAM
Ano
2015
Provas
Disciplina
Matemática
Assuntos

Avalie se as afirmativas a seguir, acerca de logaritmos, são falsas (F) ou verdadeiras (V):
- log (ab) = log (a) x log (b)

- log (a)/log (b) = log (a) - log (b)

- log (a) - log (b) = Log (a)/log (b)

As afirmativas são respectivamente:

Alternativas
Comentários
  • Alguém poderia explicar a resolução?

     

  • Propriedades dos Logarítimos.

    log a (x * y) = log a x + log a y

    log a x/y = log a x – log a y

     

  • Eu creio que o (a) e o (b) indica as bases por isso a II estaria falsa pois ela estariam em bases diferentes e a propriedade só serve para bases iguais, creio que esse é o principal motivo que fez a galera se confundir


ID
1773448
Banca
Cursiva
Órgão
CIS - AMOSC - SC
Ano
2015
Provas
Disciplina
Matemática
Assuntos

Sabendo que log2 = 0,301 e log3 = 0,477, qual o valor de log12?

Alternativas
Comentários
  • Log12= log(4.3)= log2^2 + log 3 = 2log2 + log3 = 2.0.301 + 0.477 = 1.079

  • Sabendo que log102= 0,301 e log103= 0,477

    Determine: log1012 

     

     

    log1012

    log10(2².3)/log1010

     

    log102²+log103/1

     

    2log102+log103

     

    2.0,301+0,477

     

    0,602+0,477

     

    (1,079) > RESPOSTA

     (OBS:usando as propriedades de mudanças de base.Lembrando que a base 10 estava omitida.)

     

    Bons Estudos!!!!!!!

  • log 2 = 0,301

    log 3 = 0,477

     

    log 12 = log 2.2.3

     

    log 2.2.3 = log 2 + log 2 + log 3

     

    0,301 + 0,301 + 0,477 = 1,079

  • log12

    log(4.3)

    log4+log3

    log2^2+log3

    2.log2+log3

    2.0,301+0,477

    0,602+0,477=1,079 !

  • Log12 = Log 4*3

    Log (2)² *+ log 3

    2log 2+ log 3

    2(0,301)+(0,477) = 1,079

  • LOG12= log2² x log3

    Uma vez que, 2x2x3= 12

    Seguindo:

    Log12= 2xlog2 + log3

    Log12= 2x0,301+ 0,477

    Log12= 0,602+ 0,477

    Log12= 1,079


ID
1773847
Banca
Aeronáutica
Órgão
ITA
Ano
2015
Provas
Disciplina
Matemática
Assuntos

Seja (a1, a2, a3,...) a sequência definida da seguinte forma: a1 = 1000 e an = log10(1 + an-1) para n > 2. Considere as afirmações a seguir: 

I. A sequência (an) é decrescente.

II. an > 0 para todo n > 1.

III. an < 1 para todo n > 3.

É (são) verdadeira(s)


Alternativas
Comentários
  • Temos:

    a₁ = 1000

    aₙ = log₁₀(1 + aₙ₋₁) sendo n≥2

    Resolução!!

    a₂ = log₁₀(1 + a₂₋₁) ⇒ a₂ = log₁₀(1 + a₁) ⇒ a₂ = log₁₀(1001) ⇒ a₂ = log₁₀(1001) ⇒ a₂ ≈ 3

    a₃ = log₁₀(1 + a₃₋₁) ⇒ a₃ = log₁₀(1 + a₂) ⇒ a₃ = log₁₀(1+3) ⇒ a₃ = log₁₀(4) ⇒ a₃ ≈ 0,6

    a₄ = log₁₀(1 + a₄₋₁) ⇒ a₄ = log₁₀(1 + a₃) ⇒ a₄ = log₁₀(1+0,6) ⇒ a₄ = log₁₀(1,6) ⇒ a₄ ≈ 0,2

    I. A sequência (an) é decrescente.  V

    II. an > 0 para todo n > 1. V

    III. an < 1 para todo n > 3. V

    Letra D de Jeans


ID
1785742
Banca
Marinha
Órgão
CAP
Ano
2015
Provas
Disciplina
Matemática
Assuntos

Coloque F (falso) ou V (verdadeiro) nas afirmativas abaixo, com relação ao conceito de logaritmo e assinale a opção correta.

( ) Se 3 -3 = 1/27, então -3 é o logaritmo de 1/27 na base 3.

( ) Se (0,9)2 = 0,81; então 2 .0,81 é o logaritmo de 0,9 na base 2.

( ) Se 64°'5 = 8, então 8 é o logaritmo de 64 na base 0,5.

( ) Se 27 = 128, então 7 é o logaritmo de 128 na base 2 .

Alternativas
Comentários
  • Só seguir as regras das propriedades e substituir os valores pelas letras.


ID
1825927
Banca
CONSULPLAN
Órgão
CBM-PA
Ano
2016
Provas
Disciplina
Matemática
Assuntos

A diferença entre os logaritmos de dois número, X e Y, na base 2 é 5. Sabendo que a soma de X e Y é 99 e que X > Y, então, na equação log(x–66)T = Y – 1 , T é igual a:

Alternativas
Comentários
  •   x+y=99

    Log2X - Log2Y= 5   Log2(X/Y)= 5   2^5=X/Y   X+Y=99    Logo, X=96 e Y=3          Log(x-66)T = y-1     Log30T=2  30^2=T       T=900              Bons Estudos
  • log2^x - log2^y= 5

    log2^ x/y= 5

    x/y=2^5

    x/y=32

    ´´Sabendo que a soma de X e Y é 99´´

    ou seja;

    x+ y=99

    desenvolvendo--- x= 99-y

    jogando na primeira equação:x/y=32

    99-y/y=32

    99-y= 32y

    99=33y

    y= 3

    logo, x é igual a 99-3=96.

    portanto: log(x–66)T = Y – 1---->>>  log 30^t= 2

    ->> T= 30^2= 900

    espero ter ajudado!


ID
1846486
Banca
CONSULPLAN
Órgão
CBM-PA
Ano
2016
Provas
Disciplina
Matemática
Assuntos

“O domínio da função f(x) = log5(2x – 4) é D(f) = {x ∈ R ǀ  __________}." Assinale a alternativa que completacorretamente a afirmativa anterior.

Alternativas
Comentários
  • Sabemos pela definição de logaritmo que:

    ax = b ⇔ x = loga(b), sendo b > 0, a > 0 e a ≠ 1.

    Assim, (2x - 4) > 0. Logo o domínio da função f(x) será:

    2x - 4 > 0
    2x > 4
    x > 2


    Resposta: Alternativa A.
  • Muito fácil, acompanhem:

    2x - 4 > 0

    x > 4/2

    x > 2

  • Questão com inequação do 1 grau e um conceito especifico de Logaritmo

    >>O logaritmando do logaritmo tem de ser maior que 0

    então a sua condição para ser função logarítmica é B>0

    log ( 2x-4) logaritmando

    5

    Coloque a condição Logaritmando >0

    2x-4>0

    2x>4

    x>2

    LETRA A

    APMBB

  • Jurisprudência do quê? kkkkkkkk

  • Fácil demais! é só você ter em mente as condições de existência, onde (A>0, b>0 e b diferente de 1)

    Então, teremos para a :

    2x-4>0

    2x>4

    x>4/2

    x>2


ID
1850308
Banca
FATEC
Órgão
FATEC
Ano
2011
Provas
Disciplina
Matemática
Assuntos

Considere a equação polinomial x3 − 9x2 + kx + 21 = 0, com k real. Se suas raízes estão em progressão aritmética, o valor de log2 (3k − 1)2 é

Alternativas
Comentários

ID
1862983
Banca
UFMT
Órgão
TJ-MT
Ano
2016
Provas
Disciplina
Matemática
Assuntos

Sobre propriedades de logaritmos, marque V para as verdadeiras e F para as falsas.

( ) Sendo a, b e c números reais positivos, a ≠ 1, então: logaa (b ⋅ c) = loga - loga c.
( ) Sendo a e b números reais positivos, a ≠ 1, e m um número real então: loga bm = mloga b.
( ) Sendo a, b e c números reais positivos, a ≠ 1, então: loga  (b/c) = loga b + loga c .

Assinale a sequência correta.

Alternativas
Comentários
  • (f) Sendo a, b e c números reais positivos, a ≠ 1, então: logaa (b ⋅ c) = loga b - loga c.( essa propriedade se chama logaritmo do produto

    EX. logaa (b ⋅ c)= log b + log c  (soma e não subtrai igual a cima)

     

    (v) Sendo a e b números reais positivos, a ≠ 1, e m um número real então: loga bm = mloga b.( essa propriedade se chama logaritmo da potência) onde o expoente passa para o lado esquerdo e multiplica o logaritmo.

     

    (f) Sendo a, b e c números reais positivos, a ≠ 1, então: loga  (b/c) = loga b + loga c .( essa propriedade se chama logaritmo do quociente)

    EX. loga  (b/c)= log b log c (subtrai e não soma igual a cima)

  • Revisãozinha de propriedades de Logaritmos.


ID
1910845
Banca
Exército
Órgão
EsSA
Ano
2015
Provas
Disciplina
Matemática
Assuntos

Dados log3 = α e log2 = b, a solução de 4x = 30 é

Alternativas
Comentários
  • 4^x = 30

    Log4^x = Log30

    Log2^2x = Log (3.10)

    2xLog2 = Log3 + Log10

    2x.b = a + 1

    x = a+1/2b

  • 4^x=30

    Log 2^²x = log (3.10)

    2xlog2 = a+1 , [pois log de 10 na base 10 é 1]

    2bx= a+1

    x= a+1/2b

    LETRA D

    APMBB

  • Foi passada a seguinte equação 4^x = 30.

    Você faz o contrário da regra da bundinha: Log4^x = Log30

    Simplifica a vida: 4 vira 2² e 30 vira 10 • 3.

    Então fica Log2²^x = log10 • 3

    Pela regra do chute você joga o 2^x pra trás do Log, ficado 2x • log2 = log 10 • 3 ----> log 2 = B

    2xB = log10 • 3 -----> log10 = 1

    2xB = 1 + log 3 -----log 3 = A

    2xB = 1 + A ----> Você sempre quer encontrar o valor de x na matemática

    x = 1 + A / 2b

    Letra D de Deu bom


ID
1933600
Banca
Marinha
Órgão
EFOMM
Ano
2014
Provas
Disciplina
Matemática
Assuntos

Os números reais positivos a1, a2,...., an formam, nessa ordem, uma progressão geométrica de razão q . Nesse caso, é correto afirmar que a sequência loga1, loga2 ,..... ,logan forma

Alternativas
Comentários
  • Excelente

  • a1, a2, a3 ---> PG com razão q

    a2 = a1.q ---> a3 = a1.q²

    loga1, loga2, loga3 ---> 

    log(a1) , log(a1.q) , log(a1.q²)

    log(a1) , loga1 + log(q) , log(a1) + log(q²)

    log(a1) , loga1 + log(q) , log(a1) + 2.log(q)

    Note que a última linha é uma progressão aritmética de razão r = log(q)

    Se q > 1 ---> log(q) > 0 ---> PA crescente

    Se q = 1 ---> log(q) = 0 ---> PA com todos os termos iguais

    Se 0 < q < 1 ---> log(q) < 0 ---> PA decrescente

    Referência: https://pir2.forumeiros.com/t118147-questao-de-pg


ID
1935571
Banca
Marinha
Órgão
EFOMM
Ano
2009
Provas
Disciplina
Matemática
Assuntos

Sabendo que o log30 3 = a e log30 5 = b, que opção representa log10 2 ?

Alternativas
Comentários
  • Log 2 na base 10 é igual log 2 na base 30 dividido por log 10 na base 30.

    Log de 2 na base 30 é igual log de 30/15 na base 30.

    Log de 10 na base 30 é igual log 30/3 na base 30.

    Só faz as operações logarítmicas e substitui.

  • https://brainly.com.br/tarefa/1134554

    resposta da Niiya está muito completa!


ID
1939714
Banca
Exército
Órgão
EsPCEx
Ano
2009
Provas
Disciplina
Matemática
Assuntos

Sabendo-se que logx + logx3 + logx5+...+logx199=10000, podemos afirmar que x pertence ao intervalo

Alternativas
Comentários
  • x=10

  • Boa questão, tem que usar soma de P.A e saber o número de termos dessa P.A

  • Note que os termos formam uma P.A. Precisamos, primeiramente, descobrir o número de termos dessa P.A.

    An = A1 +(N - 1).R

    199 = 1 + (N - 1).2

    199 = 1 + 2N -2

    200 = 2N

    N = 100 termos

    Agora precisamos saber a soma de todos esses termos.

    Sn = N(A1 + An)/2

    Sn = 100(1 + 199)/2

    Sn = 100.200/2

    Sn = 10000

    Logo:

    10000.LOG x = 10000

    LOG x = 1

    10¹ = x

    x = 10

    GABARITO: LETRA E


ID
1942168
Banca
CETREDE
Órgão
Prefeitura de Caucaia - CE
Ano
2016
Provas
Disciplina
Matemática
Assuntos

Qual o valor de x na equação log (2x + 6) = 2?

Alternativas
Comentários
  • Sabendo que a base do logaritmo comum é 10, temos:

    log (2x + 6) = 2
    Onde:

    base = 10
    logaritmo = 2
    logaritmando = (2x + 6).

    Aplicando as propriedades de logaritmo:

    base^(logaritmo) = logaritmando
    10^(2) = 2x + 6
    100 = 2x + 6
    2x = 94

    x = 47. ALTERNATIVA A.

  • como a base não está explícita, então ela vale 10.
    sabemos que 10ˆ2= 2x+6

    logo temos, 100=2x+6

    agora valos tentar isolar o x

    2x=100-6

    2x=94

    x=94/2     x=47

    abracos!

  • log10(2x+6)=2   

    log(2x+6)=10²

    (2x+6)=100

    2x= 100-6

    2x=94

    x=94/2

    x=47

    ----------------------------------------------------------------------

    OBS: Aquela base 10 está omitida, sempre que aparecer a base "vazia" ou "sem base" significa que a base é 10.

     

    BONS ESTUDOS!!!!!!!

  • Aplicando a propriedade na qual a base do log pega o logaritimo como expoente e o logaritimando como resultado entao temos ...obs:log com base "vazia"=10

    10^2=2x+6

    100=2x+6

    100-6=2x

    94=2x

    X=94÷2

    X=47........gabarito letra A

  • Alguém poderia me explicar, por favor,  pq o logaritimo é 10 e o logaritimando é 2, e pq foi elevado 10ˆ2? Não consegui entender essa propriedade.

  • 10^2=2x+6

    100=2x+6

    2x+100-6

    2x=94

    x=94/2

    x=47 !

    OBS:sempre que não estiver mostrando a base ela e 10 !!!

  • 10^2x+6=2 logo 10^2= 2x+6

    10^2=100 dai substituindo fica 100=2x+6

    logo 2x=100-6

    2x=94

    x=94/2

    x=47

    ABRAÇOS!

  • Log (2x+6) = 2

    10² = 2x+6

    100 = 2x+6

    100 - 6 = 2x

    94 = 2x

    x = 94/2

    x = 47

    OGMO/ES - 2022

  • Log (2x+6) = 2

    10² = 2x+6

    100 = 2x+6

    100 - 6 = 2x

    94 = 2x

    x = 94/2

    x = 47

    OGMO/ES - 2022


ID
1958818
Banca
Aeronáutica
Órgão
EEAR
Ano
2016
Provas
Disciplina
Matemática
Assuntos

Se log 2 = 0,3 e log 36 = 1,6, então log 3 = _____.

Alternativas
Comentários
  • Fatora 36 = 2 ao quadrado vezes 3 ao quadrado.

    Log2 ao quadrado mais log3 ao quadrado é igual a 1,6.

    2vezes log2 + 2vezes log3 = 1,6

    2× 0,3 + 2log3 = 1,6

    0,6 + 2log3 = 1,6

    2log3 = 1,6 - 0,6

    Log3 = 1/ 2

    Log3 = 0,5

  • Log 3 = ?

    Log 2 = 0,3

    Log 36 = 1,6

     

    Log 36 pode ser reescrito desta forma:  Log 6² ------->  2Log 6 = 1,6

     

    2Log 6 = 1,6

    Log 6 = 1,6/2

    Log 6 = 0,8

     

    Usando a propriedade do quociente, temos:

    Log a/c = Log a - Log c

    Log 6/2 = Log 6 - Log 2

     

    Log 6 - Log 2 = 0,8 - 0,3 =  0,5

     

    Gabarito B


ID
1978378
Banca
Aeronáutica
Órgão
AFA
Ano
2010
Provas
Disciplina
Matemática
Assuntos

Um médico, apreciador de logaritmos, prescreveu um medicamento a um de seus pacientes, também apreciador de logaritmo, conforme a seguir.  

Tomar x gotas do medicamento α de 8 em 8 horas.

A quantidade de gotas y diária deverá ser calculada pela fórmula log8 y = log2 6

Considerando log 2 = 3/10  e log 3 = 0,48 , é correto afirmar que log2 x é um número do intervalo  

Alternativas
Comentários
  • A função dada pelo médico é:

    log₈ y = log₂ 6

    Podemos reescrever esta função como:

    log₂.₂.₂ y = log₂ (2*3)

    Temos as seguintes propriedades:

    logxⁿ y = 1/n logx y

    log (x.y) = log x + log y

    Escrevemos então:

    1/3 log₂ y = log₂ 2 + log₂ 3

    Aplicando a mudança de base, temos:

    1/3 log y = log 2/log 2 + log 3/log 2

    1/3 log₂ y = 1 + 0,48/0,3

    log₂ y = 3(1+1,6)

    log₂ y = 7,8

    y = 2^8 Podemos arredondar para 8 para ficar mais fácil a resolução

    y = 256

    Como y é a quantidade diária de gotas e ele quer o X que são 3 por dia (visto que 8 em 8 horas, 3 em 24 horas)

    x = 256/3

    x = 85 aproximadamente

    Calculando ( eu tive que lembrar de um exercício que eu fiz anteriormente que usou Log 17= 1,2 )

    Infelizmente terá que saber para fazer na mão....

    log₂ 85 ~ 6,22

    Resposta: A

  • X gotas de 8 em 8 horas → Logo, você pegará o valor total diário (y) e dividirá por 3 para descobrir X.

    log₈ y=log₂ 6

    y = 8^ (log₂6)

    y = 2^3(log₂6)

    y= 2^(log₂6^3)

    y = 6^3

    y = 216

    ...

    216/3 → 72 = x

    log₂ x = log ₂ 72......... agora é só fatorar 72, depois mudar a base para 10... → 6,2.

    D


ID
1979110
Banca
Aeronáutica
Órgão
EEAR
Ano
2009
Provas
Disciplina
Matemática
Assuntos

Considerando n > 1, se loga n = n, então o valor de a é

Alternativas
Comentários
  • A^n=n -> extrai a raiz n dos dois lados. A=raiz n de n, usando a propriedade raiz x de y como raiz y^1/x temos : n^1/n (o radical era n e o radicado também)
  • Substitua N por qualquer número maior que 1 que a resolução se torna bem mais fácil...


ID
2000305
Banca
Aeronáutica
Órgão
EEAR
Ano
2009
Provas
Disciplina
Matemática
Assuntos

Considerando n > 1, se loga n = n, então o valor de a é

Alternativas
Comentários
  • Para tornar mais simples e conferir e provar o resultado, substitua N por um valor e teste as opções de resposta


    Resposta: Letra D

  • log a (N)= N

    a^n=n

    a=raiz(n) de n = n^1/n

  • chama n de um número qqr maior que 1 e faça a conta, aí depois é só substituir. facilita mt!

  • http://sketchtoy.com/69758411

  • boa kleber


ID
2003989
Banca
Aeronáutica
Órgão
EEAR
Ano
2012
Provas
Disciplina
Matemática
Assuntos

Se log x + log y = k, então log x5 + log y5 é

Alternativas
Comentários
  • log x^5 + log y^5

    5 * log x + 5 * log y 

    5 ( log x + log y )

    Sabendo que log x + log y = k

    Resposta : 5k

  • excelente. mas se tivesse um 25k nas respostas eu iria cair fácil. garotiei


ID
2004337
Banca
Aeronáutica
Órgão
EEAR
Ano
2010
Provas
Disciplina
Matemática
Assuntos

A razão entre o logaritmo de 16 e o de 4, numa mesma base b, sendo 0 < b ≠ 1, é

Alternativas
Comentários
  • O logaritmo de 16 e de 4 podem ser escritos em base de 2. Então:

     

    Log   16 = x
          2

    2 ^ x = 16

    2 ^ x = 2 ^ 4

    x = 4

     

     

    Log   4  = y
         2

    2 ^ y = 4
    2 ^ y = 2²

    y = 2

     

     

    A razão entre Log  16 e Log 4 é:

    4/2 = 2

     

    Gabarito D

  • Fazendo de um jeito mais prático: 

    log 16=  log 2^4 = 4log 2

    log 4 = log 2^2= 2log2 

     a razão é descrita ,portanto, por  4log2/2log2 

    simplifica tendo 4/2 = 2. Gab D

  • LogA 16+ logB 4

    Base=2

    LogA 2(ELEVADO A 4) + LogB 2 ²

    LogA 4/2

    LogA:logB= 2

  • dá para perceber também que a divisão é como se fosse uma mudança de base. então log16 na base 4
  • 1) log de b na base a(qualquer letra)

    2) Antes de montarmos as contas, devemos lembrar que razão é multiplicar um número por outro.

    log de 16 na base b . log de 4 na base b

    -> Multiplição de log na mesma base pode somar a razão na mesma base, portanto:

    log de 16 na base b + log de 4 na base b

    -> Podemos fatorar 16 para que se iguale ao log de 4, portanto:

    lod 4^2 na base b + log de 4 na base b

    -> Com logs parecidos podemos multiplicar os expoentes num único b:

    log de 4^2.1 na base b

    log de 4^2 na base b

    -> Existe uma propriedade de logaritmo que indica que quando um b elevado a um expoente qualquer, o resultado do log será o próprio expoente, portanto a resposta será 2. LETRA D

    Espero que tenho ajudado!


ID
2004610
Banca
Aeronáutica
Órgão
EEAR
Ano
2010
Provas
Disciplina
Matemática
Assuntos

Sejam as funções logarítmicas f(x) = loga x e g(x) = logb x. Se f(x) é crescente e g(x) é decrescente, então

Alternativas
Comentários
  • Pra que seja crescente, é preciso que a base seja maior que um. Ou seja, b > 1. Pra ser decrescente, é preciso que seja maior que zero e menor que um, isto é, 0 < b < 1.

     

    Gabarito B

  • Todo numero entre 0 e 1 vira fração, e fração que terá o divisor sempre maior, portanto, quanto mais você elevar a x a fração, menor ela fica. Se b é decrescente, então é 0<b<1

  • PROPRIEDADE DO LOG

    logb = B>0 e A>0 e A≠1


ID
2006512
Banca
Aeronáutica
Órgão
EEAR
Ano
2012
Provas
Disciplina
Matemática
Assuntos

Para que exista a função f(x) = log (x – m), é necessário que x seja

Alternativas
Comentários
  • Da definição de logaritmo, sabemos que b > 0. Se a função é f(x) = log (x - m), então x - m é o b. Podemos dizer que:

     

    x - m > 0

    x > m

     

    Gabarito A

  • Eu fui pela lógica, log de b na base a , o x não pode ser negativo pois daria um resultado b negativo e não pode ser igual a m pois b resultaria em 0. Com essa definição já sabemos que o x não pode ser negativo e nem igual a m. Portanto:

    a) maior que m -> Essa é a resposta correta pois x é maior que m, resultando num número positivo e diferente de zero.

    b) menor que m -> lembre-se que o b não pode ser negativo.

    c) maior ou igual a m -> maior ele pode ser, pórem igual a m não, pois o resultado daria negativo.

    d) menor ou igual m -> não pode ser negativo porque o resultado de (x-m) daria negativo e b não pode ser negativo.


ID
2034610
Banca
Exército
Órgão
IME
Ano
2009
Provas
Disciplina
Matemática
Assuntos

Seja o polinômio p(x) = x3+ (ln a) x +eb, onde a e b são números reais positivos diferentes de zero. A soma dos cubos das raízes de p(x) depende

Obs.: e representa a base do logaritmo neperiano e ln a função logaritmo neperiano.

Alternativas

ID
2035549
Banca
IESES
Órgão
PM-SC
Ano
2010
Provas
Disciplina
Matemática
Assuntos

Se log9a2 - log3b = 4 então o quociente a/b vale:

Alternativas
Comentários
  • 1) Aplica a mudança de base:

    Log a^2 na base 3/ log 9 na base 3

    2) Resolve:

    Log aˆ2/9 na base 3 = Log aˆ2|3ˆ2 = Log a|3 

    Log a|3-Log b|3 =

    Log (a/b) |3 =4 =

    3ˆ4 =a/b =

    a/b = 81


ID
2042830
Banca
Aeronáutica
Órgão
FAB
Ano
2014
Provas
Disciplina
Matemática
Assuntos

Se log2 = a e log3 = b , então a solução da equação 10x = 60 é

Alternativas
Comentários
  • log 10^x = 60

    log 10^60 = x

    log60/log10 -> log 2 . 3 . 10 / log 10

    a + b + 1 / 1

    a + b+ 1 

    gabarito: B

  • 10^x = 60 

    x.log10 = log60 

    x.(1) = log(6*10) 

    x = log6 + log10 

    x =log(2*3) + 1 

    x = log2 +log3 +1 

    x = a +b +1

  • O bizu é você imaginar que 60 é 10x6 e 6=3x2...

  • A maldade da questão é fatorar o 60

    10^x=60

    10^x=2^2.3.5

    10^x=2.log2+log3+log10/5

    10^x+2a+b+1-a

    10^x=A+B+1

    B


ID
2053417
Banca
Aeronáutica
Órgão
FAB
Ano
2010
Provas
Disciplina
Matemática
Assuntos

A soma dos logaritmos de dois números positivos, na base 9, é ½. O produto desses números é

Alternativas
Comentários
  • logA na base 9 + LogB na base 9= 1/2

    Pela propriedade:  Log A + Log B = Log A.B

    Log A.B na base 9 = 1/2

    9 elevado a 1/2 = A.B

    A.B = 3

    GABARITO: A

     

     


ID
2057488
Banca
UFMT
Órgão
TJ-MT
Ano
2016
Provas
Disciplina
Matemática
Assuntos

Sobre propriedades de logaritmos, marque V para as verdadeiras e F para as falsas.

( ) Sendo a, b e c números reais positivos, a ≠ 1, então: loga(b ∙ c) = logb − logc  .

( ) Sendo a e b números reais positivos, a ≠ 1, e um número real então: logbm = m logb .

( ) Sendo a, b e c números reais positivos, a ≠ 1, então: loga (b/c) = loga b + logc .

Assinale a sequência correta.  

Alternativas
Comentários
  • O logaritmo de um produto é representado pela soma do log dos fatores, portanto, a primeira assertiva está incorreta;

    Segunda assertiva está correta, o expoente "desce" multiplicando o log.

    O logaritmo de um quociente é representado pela subtração do log, portanto, a terceira assertiva está incorreta.

    Resposta Correta = B

  • 1° =quando o logaritimando esta multiplicando log(b.c) devemos separa-los por + .....F

     

    2°O expoente do logaritimando sempre pode passar pra frente multiplicando.....V

     

    3° quando o logaritimando estiver ÷ devemos separa-los por subtraçao....F 

     

    Gabarito letra B

     

  •  log a (b.c) = log ab + log ac

    Correto

    log a (b/c) = log ab - log ac

    Bons estudos!


ID
2058013
Banca
UFMT
Órgão
TJ-MT
Ano
2016
Provas
Disciplina
Matemática
Assuntos

Sobre propriedades de logaritmos, marque V para as verdadeiras e F para as falsas.

( ) Sendo a, b e c números reais positivos, a ≠ 1, então: loga (b.c) = logab − logac.

( ) Sendo a e b números reais positivos, a ≠ 1, e m um número real então: loga bm = m loga b.

( ) Sendo a, b e c números reais positivos, a ≠ 1, então: loga (b/c)= loga b + loga c.

Assinale a sequência correta.

Alternativas
Comentários
  • A)  loga (b.c) = logab + logac. o sinal é positivo.

    B) Correto

    C) loga (b/c)= loga b - loga c o sinal é negativo.

    Qualquer coisa me corrijam por favor.


ID
2087950
Banca
Exército
Órgão
EsPCEx
Ano
2016
Provas
Disciplina
Matemática
Assuntos

O número N de bactérias de uma cultura é dado em função do tempo t (em minutos), pela fórmula N(t)=(2,5)1,2t . Considere log10 2=0,3, o tempo (em minutos) necessário para que a cultura tenha 1084 bactérias é

Alternativas
Comentários
  • N(t)=(2,5)1,2t

     

    N(t) = 10^84 , quero saber quanto vale t:

     

    log 10^84 = log (2,5) ^1,2 t

     

    84 log 10 = log (10/4) ^ 1,2 t ,  aqui está o lance da questão, transformar o 25/10 em 10/4 pra facilitar na manipulação do log.

     

    84 *1 = 1,2 t * (log 10 - log 4)

     

    84 = 1,2 t * (log10 - 2log2)

     

    84 = 1,2 t * (1 - 0,6)

     

    84 = 0,48 t

     

    t = 175

     

    GABARITO LETRA C)

     

    Edit: tinha um erro na multiplicação

     

    Bons estudos galera ..

  • é só que a resposta fica 17,5 e aletra c  esta 175 deve ta faltando uma virgula.

  • Nada a ver... Gab é 175 mesmo.

  • N(t)=(2,5)^1,2t ---> N(t)=10^84 ---> 10^84=(2,5)^1,2t ---> 2,5=5/2 ---> 10^84=(5/2)^1,2t ---> aplicar log dos dois lados ---> log10^84=log(5/2)^1,2t ---> 84log10=1,2t.(log5 - log2) ---> sabemos que log10 na base 10 é 1, e log2 na base 10 é 0,3 (conforme enunciado), então: ---> 84=1,2t.(log5 - 0,3) ---> log5 é o mesmo que log (10/2), então: log10 - log2= 1-0,3 ---> 0,7=log5 ---> 84=1,2t.(0,7 - 0,3) ---> 84=1,2t.0,4 ---> 84=0,48t ---> t=84/0,48 ---> resposta: t=175 minutos
  • Vou colocar a resolução corrida. Caso não entendam a resolução, podem entrar em contato comigo. Vamos lá:

    N(t)= 2,5^1,2t; log 2 = 3

    (2,5)^1,2t = 10^84

    (10/4) ^1,2t = 10^84;

    10^1,2t/10^84 = 4^1,2t;

    10^(1,2t - 84) = 2^2,4t;

    log2^2,4t = 1,2t - 84;

    2,4t * log2 = 1,2t - 84;

    2,4t * 0,3 = 1,2t - 84;

    0,72t = 1,2t - 84;

    0,48t =84;

    t = 84/0,48

    t = 175 min

    OUTRO JEITO DE RESOLVER A QUESTÃO:

    (2,5) ^1,2T = 10 ^ 84

    Log (2,5) ^1,2t = log 10 ^ 84;

    log (10/4) ^1,2t = 84 * log 10

    log 10 ^1,2t - log 4 ^1,2t = 84;

    1,2 t - 2,4 * log 2 = 84

    1,2 t - 0,72 t = 84

    0,48 t - 84;

    t = 175 min

    GABARITO: C

    INSTAGRAN:

    @simplificandoquestoescombizus (Jefferson Lima)

  • https://www.youtube.com/watch?v=4VXbVnCQDZk&ab_channel=Soluciona